SlideShare a Scribd company logo
1 of 44
CHAPTER 1


PARTIAL DIFFERENTIAL EQUATIONS

           A partial differential equation is an equation involving a function of two or
more variables and some of its partial derivatives. Therefore a partial differential
equation contains one dependent variable and one independent variable.
           Here z will be taken as the dependent variable and x and y the independent
variable so that z = f ( x, y ) .
           We will use the following standard notations to denote the partial derivatives.
   ∂z     ∂z   ∂2z      ∂2z      ∂2z
      = p, = q, 2 = r ,      = s, 2 = t
   ∂x     ∂y   ∂x       ∂x∂y     ∂y
           The order of partial differential equation is that of the highest order derivative
occurring in it.


Formation of partial differential equation:
          There are two methods to form a partial differential equation.
        (i) By elimination of arbitrary constants.
        (ii) By elimination of arbitrary functions.


Problems


Formation of partial differential equation by elimination of arbitrary
constants:


(1)Form the partial differential equation by eliminating the arbitrary constants

from z = ax + by + a 2 + b 2 .
Solution:



                                                                                                1
Given z = ax + by + a 2 + b 2                                ……………... (1)
         Here we have two arbitrary constants a & b.
         Differentiating equation (1) partially with respect to x and y respectively we get
         ∂z
            =a⇒ p=a                                                  ……………… (2)
         ∂x
         ∂z
            =b⇒q=a                                                   ………………. (3)
         ∂y
         Substitute (2) and (3) in (1) we get
            z = px + qy + p 2 + q 2 , which is the required partial differential equation.


(2) Form the partial differential equation by eliminating the arbitrary constants

                 x2 y2 z 2
a, b, c from       +  +    = 1.
                 a2 b2 c2
Solution:
        We note that the number of constants is more than the number of independent
         variable. Hence the order of the resulting equation will be more than 1.
                    x2 y2 z 2
                      +  +    =1                                          .................. (1)
                    a2 b2 c2
        Differentiating (1) partially with respect to x and then with respect to y, we get


                    2x 2 y
                       +   p=0                                          .....................(2)
                    a2 c2
                    2 y 2z
                       +   q=0                                          ......................(3)
                    b2 c2
        Differentiating (2) partially with respect to x,
                    1    1
                     2
                       + 2 ( zr + p 2 )                                ……………..(4)
                   a    c
                     ∂2z
            Where r = 2 .
                     ∂x
                                    c 2 zp
                                   − 2 =                                ......................(5)
                                    a     x
            From (2) and (4) ,        2
                                                                                                    .
                                    c
                                   − 2 = zr + p 2                      ......................(6)
                                    a


                                                                                                        2
From (5) and (6), we get
                                                     2
                                ∂2z    ∂z  ∂z
                              xz 2 + x  = z , which is the required partial differential
                                ∂x     ∂x  ∂x
            equation.


(3) Find the differential equation of all spheres of the same radius c having their
center on the yoz-plane.                  .
Solution:
            The equation of a sphere having its centre at ( 0, a, b ) , that lies on the yoz -plane
             and having its radius equal to c is
                                  x 2 + ( y − a ) 2 + ( z − b) 2 = c 2          ……………. (1)


            If a and b are treated as arbitrary constants, (1) represents the family of spheres
            having the given property.
            Differentiating (1) partially with respect to x and then with respect to y, we
            have
                                  2 x + 2( z − b ) p = 0                          …………… (2)
             and                  2( y − a ) + 2( z − b ) q = 0                   …………….(3)
                                                 x
              From (2),             z −b = −                                        …………….(4)
                                                 p
                                                     qx
              Using (4) in (3), y − b =                                           ……………..(5)
                                                      p
              Using (4) and (5) in (1), we get
                             q2x2 x2
                          x + 2 + 2 = c2
                              2

                              p   p

            . i.e.   (1 + p   2
                                  + q 2 ) x 2 = c 2 p 2 , which is the required partial differential
equation.


Problems




                                                                                                       3
Formation of partial differential equation by elimination of arbitrary
functions:
(1)Form the partial differential equation by eliminating the arbitrary function ‘f’
from
           z = e ay f ( x + by )


 solution: Given z = e ay f ( x + by )

            i.e.     e − ay z = f ( x + by )                            ……………(1)
            Differentiating (1) partially with respect to x and then with respect to y, we
             get
                         e − ay p = f ' ( u ).1                          ………….(2)

                         e − ay q − ae − ay z = f ' ( u ).b             …………….(3)
              where u = x + by
              Eliminating f’(u) from (2) and (3), we get
                          q − az
                                 =b
                             p
              i.e.     q = az + bp


(2) Form the partial differential equation by eliminating the arbitrary function ‘ φ ’
               x
   φ  z 2 − xy,  = 0
               z

                                       x
   Solution: Given         φ  z 2 − xy,  = 0                    ………………(1)
                                       z
                                                       x
              Let       u = z 2 − xy ,            v=
                                                       z
             Then the given equation is of the form φ ( u , v ) = 0 .
              The elimination of φ from equation (2), we get,
                                   ∂u   ∂v
                                   ∂x   ∂x = 0
                                   ∂u   ∂v
                                   ∂y   ∂y



                                                                                             4
z − px
                       2 zp − y
               i.e.                    z2 = 0
                                      − xq
                       2 zq − x
                                       z2

               i.e            ( 2 zp − y )  − xq  −  z − 2px ( 2 zq − x ) = 0
                                            2                
                                            z           z     
               i.e                px 2 − q ( xy − 2 z 2 ) = zx


(3) Form the partial differential equation by eliminating the arbitrary function ‘f’
from
          z = f ( 2 x + y ) + g ( 3x − y )

Solution: Given z = f ( 2 x + y ) + g ( 3x − y )                                    .…………(1)
              Differentiating (1) partially with respect to x,
                         p = f ′( u ).2 + g ′( v ).3                                 ………….(2)
               Where u = 2 x + y and v = 3 x − y
              Differentiating (1) partially with respect to y,
                        q = f ′( u ).1 + g ′( v ) (−1)                               …………. (3)
               Differentiating (2) partially with respect to x and then with respect to y,
                        r = f ′′( u ).4 + g ′′( v ).9                                …………. (4)
               and s = f ′′( u ).2 + g ′′( v ).(−3)                                  ………….. (5)
               Differentiating (3) partially with respect to y,
                        t = f ′′( u ).1 + g ′′( v ).1                                ………….. (6)
                Eliminating f ′′( u ) and g ′′( v ) from (4), (5) and (6) using determinants, we
                have
                                     4 9 r
                                     2 −3 s = 0
                                     1 1 t

                i.e.              5r + 5s − 30t = 0
                                  ∂2z ∂2z     ∂2z
                or                    +     −6 2 = 0
                                  ∂x 2 ∂x∂y   ∂y




                                                                                                   5
(4) Form the partial differential equation by eliminating the arbitrary function ‘ φ ’
from
           1
        z = φ ( y − x ) + φ ′( y − x ) .
           x
                           1
 Solution: Given      z = φ ( u ) + φ ′( u )                                       …………...(1)
                           x
             Where u = y − x
              Differentiating partially with respect to x and y, we get
                        1                1
                     p = φ ′( u ) (−1) − 2 φ ( u ) + φ ′′( u ) (−1)                   …………(2)
                        x               x
                       1
                    q = φ ′( u ).1 + φ ′′( u ).1                                      ………….(3)
                       x
                          1               2            2
                    r=      φ ′′( u ).1 + 2 φ ′( u ) + 3 φ ( u ) + φ ′′′( u ).1        ………..(4)
                          x              x            x
                            1             1
                    s=−       2
                                φ ′( u ) − φ ′′( u ) + φ ′′′( u ) (−1)                …………(5)
                            x             x
                       1
                    t = φ ′′( u ).1 + φ ′′′( u ).1                                    …………..(6)
                       x
            From (4) and (6), we get
                              2              2
                    r −t =      2
                                  φ ′( u ) + 3 φ ( u )
                              x             x
                              2    1                   
                          =         φ ( u ) + φ ′( u ) 
                              x2   x                   
                              2
                          =      z
                              x2
                             ∂ 2 z ∂ 2z     
             i.e.        x2  2 − 2
                             ∂x              = 2z
                                             
                                   ∂y       


Solutions of partial differential equations
       Consider the following two equations


                      z = ax + by                                                 ………..(1)




                                                                                                  6
 y
              and       z = xf                                  ………..(2)
                               x
  Equation (1) contains arbitrary constants a and b, but equation (2) contains only one
arbitrary function f.
  If we eliminate the arbitrary constants a and b from (1) we get a partial differential
equation of the form xp + yq = z . If we eliminate the arbitrary function f from (2) we get
a partial differential equation of the form xp + yq = z .
   Therefore for a given partial differential equation we may have more than one type of
solutions.


Types of solutions:


(a) A solution in which the number of arbitrary constants is equal to the number of
    independent variables is called Complete Integral (or) Complete solution.
(b) In complete integral if we give particular values to the arbitrary constants we get
    Particular Integral.
 (c) The equation which does not have any arbitrary constants is known as Singular
     Integral.


To find the general integral:
   Suppose that f ( x, y, z, p, q ) = 0                                ………....(1)
is a first order partial differential equation whose complete solution is
       φ ( x, y , z , a , b ) = 0                                       ………..(2)
Where a and b are arbitrary constants.
Let b = f ( a ) , where ‘f’is an arbitrary function.
Then (2) becomes
       φ ( x, y , z , a , f ( a ) ) = 0                                     ……….(3)
Differentiating (3) partially with respect to ‘a’, we get


       ∂φ ∂φ
         +   . f ′( a ) = 0                                                 ……….(4)
       ∂a ∂b



                                                                                              7
The eliminant of ‘a’ between the two equations (3) and (4), when it exists, is called the
general integral of (1).


Methods to solve the first order partial differential equation:
Type 1:
Equation of the form f ( p, q ) = 0                                     ………...(1)
i.e the equation contains p and q only.
Suppose that       z = ax + by + c                                      …….....(2)
is a solution of the equation


       ∂z     ∂z
          = a, = b
       ∂x     ∂y
   ⇒ p = a, q = b
substitute the above in (1), we get
       f ( a, b ) = 0

on solving this we can get b = φ ( a ) , where φ is a known function.
Using this value of b in (2), the complete solution of the given partial differential
equation is
       z = ax + φ ( a ) y + c                                   …………(3)
is a complete solution,
To find the singular solution, we have to eliminate ‘a’ and ‘c’ from
       z = ax + φ ( a ) y + c
Differentiating the above with respect to ‘a’ and ‘c’, we get
       0 = x + φ ′( a ) y ,
and 0=1.
The last equation is absurd. Hence there is no singular solution for the equation of
Type 1.


Problems:


(1) Solve     p2 + q2 = 1.


                                                                                            8
Solution:
    Given: p 2 + q 2 = 1                                        ………….(1)
    Equation (1) is of the form f ( p, q ) = 0 .
    Assume z = ax + by + c                                       ………….(2)
               be the solution of equation (1).
    From (2) we get p = a, q = b .
    (1) ⇒ a 2 + b 2 = 1

         ⇒ b = ± 1− a2                                            ……….(3)
     Substitute (3) in (2) we get

            z = ax ± 1 − a 2 y + c                                 ……....(4)

     This is a complete solution.


    To find the general solution:
            We put c = f ( a ) in (4),where ‘f’ is an arbitrary function.


      i.e. z = ax ± 1 − a 2 y + f ( a )                        …………(5)

     Differentiating (5) partially with respect to ‘a’, we get


                    a
             x              y + f ′( a ) = 0                ……………(6)
                  1− a2
    Eliminating ‘a’ between equations (5) and (6), we get the required general solution.


    To find the singular solution:
    Differentiate (4) partially with respect to ‘a’ and ‘c’, we get
                         a
            0= x±               ,
                        1− a2
            0=1.(which is absurd)
    so there is no singular solution.


(2) Solve p + q = pq



                                                                                           9
Solution:
    Given: p + q = pq                                       ………..(1)
    Equation (1) is of the form f ( p, q ) = 0
    Assume z = ax + by + c                                      ………(2)
             be the solution of equation (1).
    From (2) we get p = a, q = b
    (1) ⇒ a + b = ab
                   a
        ⇒b=                                                     …….....(3)
                 a −1
    Substituting (3) in (2), we get
                      a
         z = ax +        y+c                                    ………(4)
                    a −1
    This is a complete solution.


    To find the general solution:
        We put c = f ( a ) in (4), we get
                               a
            z = ax +              y + f ( a)                     ……..(5)
                             a −1
    Differentiating (5) partially with respect to ‘a’, we get


                     1
            x−                 y + f ′( a )                 ………..(6)
                 ( a − 1) 2
    Eliminating ‘a’ between equations (5) and (6), we get the required general solution


    To find the singular solution:
    Differentiating (4) with respect to ‘a’ and ‘c’.
                         1
         0= x−                   y,
                  ( a − 1) 2
    and 0=1 (which is absurd).
   So there is no singular solution.




                                                                                          10
Type 2: (Clairaut’s type)
The equation of the form
             z = px + qy + f ( p, q )                          ………(1)
is known as Clairaut’s equation.


Assume       z = ax + by + c                                  …………(2)
be a solution of (1).
            ∂z     ∂z
               = a, = b
            ∂x     ∂y
          ⇒ p = a, q = b
Substitute the above in (1), we get
             z = ax + by + f (a, b)                           ………..(3)
which is the complete solution.


Problem:


(1) Solve z = px + qy +        pq

Solution:
    Given: z = px + qy +        pq                            ……….(1)
    Equation (1) is a Clairaut’s equation
    Let z = ax + by + c                                       ………..(2)
    be the solution of (1).
    Put p = a, q = b in (1), we get

         z = ax + by + ab                                     ……….(3)
    which is a complete solution.


    To find the general solution:
         We put b = f ( a ) in (3), we get

        z = ax + f ( a ) y + af ( a )                         ………(4)
    Differentiate (4) partially with respect to ‘a’, we get



                                                                         11
1
          0 = x + f ′( a ) y +                  [ af ′( a ) + f ( a ) ]    ………..(5)
                                   2 af ( a )

    Eliminating ‘a’ between equations (4) and (5), we get the required general solution


    To find singular solution,
    Differentiate (3) partially with respect to ‘a’, we get
                          1
            0= x+                 .b
                     2 ab

                              b
            ⇒x=−                                                          ………..(6)
                         2 a
    Differentiate (3) partially with respect to ‘b’, we get
                          1
            0= y+                 .a
                         2 ab

                              a
            ⇒ y=−                                                         ………..(7)
                         2 b
    Multiplying equation (6) and (7),we get
                    b      a 
            xy =  −          
                  2 a  − 2 b 
                             
                     1
              xy =
                     4
             4 xy = 1
                                       q
(2) Solve      z = px + qy +             −p
                                       p
Solution:
                                       q
    Given: z = px + qy +                 −p                               ……….(1)
                                       p
    Equation (1) is a Clairaut’s equation
    Let      z = ax + by + c                                              ………...(2)
    be the solution of (1).
    Put p = a, q = b in (1), we get
                                  b
             z = ax + by +          −a                                     ……….(3)
                                  a


                                                                                          12
which is the complete solution.


To find the general solution:
    We put b = f ( a ) in (3), we get
                                f ( a)
        z = ax + f ( a ) y +           −a                     ……..(4)
                                  a
Differentiate (4) partially with respect to ‘a’, we get
                              af ′( a ) − f ( a )
       0 = x + f ′( a ) y +                       −1          ……..(5)
                                       a
Eliminating ‘a’ between equations (4) and (5), we get the required general
solution


To find the singular solution:
Differentiate (3) partially with respect to ‘a’,
                    b
        0= x−          −1
                    a2
              b
    ⇒x=          +1
              a2
    ⇒ a2x = b +1
    ⇒ b = a2 x −1                                          .............. (4)
Differentiate (3) partially with respect to ‘b’,
                    1
        0= y+
                    a
                1
    ⇒a=−                                                   ………….(5)
                y
Substituting equation (4) and (5) in equation (3), we get


          −1
     z =   x + ( a 2 x − 1) y +
                                  ( a 2 x − 1) −  − 1 
                                                  
          y                          −1         y 
                                                
                                        y
           −x                             1
     z=       + a 2 yx − y − a 2 xy + y +
            y                             y




                                                                                13
−x 1
          z=          +
                    y   y
              yz = 1 − x


Type 3:
Equations not containing x and y explicitly, i.e. equations of the form
              f ( z , p, q ) = 0                                 ……….(1)
For equations of this type ,it is known that a solution will be of the form
          z = φ ( x + ay )                                      ……….(2)
Where ‘a’ is the arbitrary constant and φ is a specific function to be found out.
Putting x + ay = u , (2) becomes z = φ ( u ) or z ( u )
                  dz ∂u dz
  ∴       p=        .  =
                  du ∂x du
                   dz ∂u    dz
and       q=         .   =a
                   du ∂y    du
If (2) is to be a solution of (1), the values of p and q obtained should satisfy (1).
                    dz dz 
       i.e.      f  z, , a  = 0                                 ……..(3)
                    du du 
From (3), we get
               dz
                  = ψ ( z, a )                                  ……….(4)
               du
Now (4) is a ordinary differential equation, which can be solved by variable separable
method.
The solution of (4), which will be of the form g ( z, a ) = u + b or g ( z, a ) x + ay + b , is the
complete solution of (1).
The general and singular solution of (1) can be found out by usual method.


Problems:


(1)Solve z = p 2 + q 2 .
Solution:



                                                                                                      14
Given: z = p 2 + q 2                                                   …………(1)

    Equation (1) is of the form f ( z , p, q ) = 0
 Assume z = φ ( u ) where, u = x + ay be a solution of (1).
       ∂z dz ∂u dz                                           dz
  p=     =  .  =                                     ⇒ p=                …….(2)
       ∂x du ∂x du                                           du
       ∂z dz ∂u    dz                                             dz
  q=     =  .   =a                                       ⇒q=a            ……(3)
       ∂y du ∂y    du                                             du
 Substituting equation (2) & (3) in (1), we get


                       2                    2
       dz    dz 
  z =   + a2 
       du    du 
                       2

  z =   (1 + a 2 )
       dz 
       du 
                   2
     dz    z
      =
     du  1+ a2
                                 1
           dz    z2
              =
           du (1 + a 2 )
   By variable separable method,
       dz                     du
                   =
                           (1 + a )
           1                       1
                                 2 2
       z   2


    By integrating, we get
           dz                   1
       ∫               =                   ∫ du + c
                           (1 + a )
               1                      1
                                    2 3
           z   2



           2 z=
                               ( x + ay )           +c
                              (1 + a )
                                         1
                                       2 2




                       z=
                                ( x + ay )           +
                                                         c

                               2(1 + a          )
                                              1
                                            2 2          2


               z=
                            ( x + ay )      +k                            ……….(4)
                           2(1 + a     )
                                        1
                                      2 2




                                                                                    15
This is the complete solution.


         To find the general solution:
                  We put k = f ( a ) in (4), we get

                   z=
                         ( x + ay )        + f ( a)                 ……..(5)
                        2(1 + a        )
                                    1
                                  2 2



          Differentiate (5) partially with respect to ‘a’, we get
                          y − xa
                   0=                      + f ′( a )             ………..(6)
                        2(1 + a        )
                                    3
                                  2 2




         Eliminating ‘a’ between equations (4) and (5), we get the required general
         solution.


         To find the singular solution:
         Differentiate (4) partially with respect to ‘a’ and ‘k’, we get
                          y − xa
                  0=                                              ………..(7)
                        2(1 + a    )
                                    3
                                  2 2



         and         0 = 1 (which is absurd)
          So there is no singular solution.


(2)Solve 9( p 2 z + q 2 ) = 4 .
Solution:
           Given: 9( p 2 z + q 2 ) = 4                              ………(1)

           Equation (1) is of the form f ( z , p, q ) = 0
           Assume z = φ ( u ) where , u = x + ay be a solution of (1).
                 ∂z dz ∂u dz                            dz
            p=     =  .  =                       ⇒ p=                …….(2)
                 ∂x du ∂x du                            du


                 ∂z dz ∂u    dz                              dz
            q=     =  .   =a                       ⇒q=a              ……(3)
                 ∂y du ∂y    du                              du



                                                                                      16
Substituting equation (2) & (3) in (1), we get
   dz  2         2
            2  dz  
                      
9   z + a   = 4
   du       du  
                     
            2
   dz 
                [
 9  z + a 2 = 4             ]
   du 
        2
  dz      4
   =
  du  9 z + a2      [             ]
  dz   2   1
     =± .
  du   3 z + a2

                  2
    z + a 2 dz = ± du
                  3
 Integrating the above, we get

  ∫ (z + a )
                  1
                                    2
                                    3∫
                2 2
                      dz = ±           du + c


    (z + a2 ) 2 = ± 2 u + c
              3
  2
  3                 3

     ( z + a 2 ) 2 = ± 2 ( x + ay ) + c
                 3
   2
   3                   3

  ⇒ ( z + a 2 ) 2 = ±( x + ay ) + k
                          3
                                                              ………..(4)

  This is the complete solution.


   To find the general solution:
                    We put k = f ( a ) in (4), we get

                (z + a )
                                3
                              2 2
                                    = ±( x + ay ) + f ( a )        ……..(5)

  Differentiate (5) partially with respect to ‘a’, we get

             − 3a ( z − a 2 ) 2 = ± y + f ′( a )
                                        1
                                                                   ……..(6)

  Eliminating ‘a’ between equations (4) and (5), we get the required general
  solution.
      To find the singular solution:
  Differentiate (4) partially with respect to ‘a’ and ‘k’, we get



                                                                               17
3a ( z + a 2 ) 2 = ± y
                          1
                                                                    ………..(7)

               and        0 = 1 (which is absurd)
               So there is no singular solution.


Type 4:
Equations of the form
                              f ( x, p ) = g ( y , q )                      ………..(1)
i.e. equation which do not contain z explicitly and in which terms containing p and x can
be separated from those containing q and y.
To find the complete solution of (1),
    We assume that f ( x, p ) = g ( y , q ) = a .where ‘a’ is an arbitrary constant.
Solving f ( x, p ) = a ,we can get p = φ ( x, a ) and solving g ( y , q ) = a ,we can get

q = ψ ( y, a ) .
Now
               ∂z     ∂z
        dz =      dx + dy or pdx + qdy
               ∂x     ∂y

  i.e. dz = φ ( x, a ) dx + ψ ( y, a ) dy
Integrating with respect to the concerned variables, we get
         z = ∫ φ ( x, a )dx + ∫ψ ( y, a ) dy + b                            ……….(2)

The complete solution of (1) is given by (2), which contains two arbitrary constants ‘a’
and ‘b’.
The general and singular solution of (1) can be found out by usual method.


Problems:
(1)Solve pq = xy .
Solution:
       Given: pq = xy
                   p y
             ⇒      =                                                ………..(1)
                   x q

       Equation (1) is of the form f ( x, p ) = g ( y , q )



                                                                                            18
p y
      Let           = = a (say)
                   x q
                  p
                    =a      ⇒ p = ax                            ………….(2)
                  x
                  y                  y
    Similarly,      =a      ⇒q=                                 …………(3)
                  q                  a
     Assume dz = pdx + qdy be a solution of (1)
     Substitute equation (2) and (3) to the above, we get
                                y
                 dz = axdx +      dy
                                a
     Integrating the above we get,
                                    1
                 ∫ dz = a ∫ xdx + a ∫ ydy + c
                     ax 2 y 2
                  z=     +    +c
                      2    2a
                             y2
                  2 z = ax +2
                                +k                              ………..(4)
                             a
      This is the complete solution.
      The general and singular solution of (1) can be found out by usual method.


(2) Solve p + q = x + y .
  Solution:
     Given: p + q = x + y
             ⇒p−x =q− y                                       ………….. (1)
       Equation (1) is of the form f ( x, p ) = g ( y , q )
       Let        p − x = y − q = a (say)
                     p−x =a        ⇒ p = x+a                  …………. (2)
       Similarly, y − q = a        ⇒ q = y−a                  ……………(3)
       Assume dz = pdx + qdy be a solution of (1)
       Substitute equation (2) and (3) to the above, we get
                 dz = ( x + a ) dx + ( y − a ) dy
       Integrating the above we get,



                                                                                   19
∫ dz = ∫ ( x + a ) dx + ∫ ( y − a ) dy + c
                   z=
                        (x   2
                                 + y2 )
                                        + a( x − y ) + c
                                 2
                    2 z = x 2 + y 2 + 2a ( x − y ) + c              ……………(4)
        This is the complete solution.
         The general and singular solution of (1) can be found out by usual method.


Equations reducible to standard types-transformations:
Type A:
Equations of the form f ( x m p, y n q ) = 0 or            f ( x m p, y n q, z ) = 0 .
Where m and n are constants, each not equal to 1.
We make the transformations x 1− m = X and y 1− n = Y .
                 ∂z ∂z ∂X                               ∂z
  Then      p=     =  .   = (1 − m ) x − m P, where P ≡    and
                 ∂x ∂X ∂x                               ∂X


               ∂z ∂z ∂Y                              ∂z
          q=     =  .   = (1 − n ) y −n Q, where Q ≡
               ∂y ∂Y ∂y                              ∂Y

Therefore the equation f ( x m p, y n q ) = 0 reduces to f { (1 − m ) P, (1 − n ) Q} = 0, .which is a
type 1 equation.
The equation f ( x m p, y n q, z ) = 0 reduces to f { (1 − m ) P, (1 − n ) Q, z} = 0, .which is a type 3
equation.


Problem:
(1)Solve p 2 x 4 + y 2 zq = 2z 2 .
Solution:
     Given: p 2 x 4 + y 2 zq = 2z 2
     This can be written as

                    ( px ) + ( qy ) z = 2z .
                             2 2            2       2



      Which is of the form f ( x p, y q, z ) = 0 , where m=2,n=2.
                                        m       n




                                                                                                     20
1− m      1                1
      Put X = x =          ; Y = y 1− n =
                         x                y
               ∂z ∂z ∂x
          P=     = .    = p ( − x 2 ) = − px 2
               ∂X ∂x ∂X
               ∂z ∂z ∂y
          Q=     = .    = q ( − y 2 ) = −qy 2
               ∂Y ∂y ∂Y
     Substituting in the given equation,
             P 2 − qz = 2z 2 .

     This is of the form f ( p, q, z ) = 0 .
         Let Z = f ( X + aY ) , where u = X + aY
                 dz      dz
            P=      ,Q=a
                 du      du
      Equation becomes,          .
                                          2
                                      dz    dz
                                       − az    − 2z 2 = 0
                                      du    du
                    dz               dz az ± a 2 z 2 + 8 z 2
      Solving for      ,                =
                    du               du        2
                                     dz a ± a 2 + 8
                                        =           du
                                      z     2

                                              a ± a2 + 8
                                 log z =                 ( X + aY ) + b
                                                  2

                                         a ± a2 + 8  1 a 
                                 log z =             +  + b is a complete solution.
                                                    x y
                                             2           
       The general and singular solution can be found out by usual method.


Type B:
Equations of the form f ( z k p, z k q ) = 0 or          f ( z k p , z k q, x, y ) = 0 .
Where k is a constant, which is not equal to -1.
We make the transformations Z = z k +1 .
                ∂Z
  Then    P=       = ( k + 1) z k p and
                ∂x




                                                                                           21
∂Z
          Q=       = ( k + 1) z k p
                ∂y

                                                            P     Q 
Therefore the equation f ( z k p, z k q ) = 0 reduces to f      ,      = 0, which is a type 1
                                                            k +1 k + 1
equation.
                                                              P                
                   f ( z k p, z k q, x, y ) = 0 reduces to f 
                                                                     Q
The equation                                                       ,     , x, y  = 0, which is a
                                                              k +1 k +1        
type 4 equation.


Problems:
(1)Solve: z 4 q 2 − z 2 p = 1.
Solution:
     Given: z 4 q 2 − z 2 p = 1.

     The equation can be rewritten as ( z 2 q ) − ( z 2 p ) = 1
                                                  2
                                                                             ………(1)

     Which contains z 2 p and z 2 q .
      Hence we make the transformation Z = z 3
                      ∂Z
                   P=     = 3z 2 p
                      ∂x
                           P
                  ⇒ z2 p =
                           3
                          Q
      Similarly z q =
                 2

                          3
       Using these values in (1), we get
                   Q 2 − 3P = 9                                           ………..(2)
        As (2) is an equation containing P and Q only, a solution of (2) will be of the form
                    Z = ax + by + c                                     ………….(3)
        Now P = a and Q = b, obtained from (3) satisfy equation (2)
                    b 2 − 3a = 9
              i.e. b = ± 3a + 9

        Therefore the complete solution of (2) is Z = ax ± 3a + 9 y + c

        i.e complete solution of (1) is z 3 = ax ± 3a + 9 y + c


                                                                                                    22
Singular solution does not exist. General solution is found out as usual.
Type C:
Equations of the form f ( x m z k p, y n z k q ) = 0 , where           m, n ≠ 1; k ≠ −1

We make the transformations
                  X = x 1− m , Y = y 1−n and Z = z k +1
                          ∂Z dZ ∂z dx
     Then          P=        =   . .
                          ∂X   dz ∂x dX
                                           xm
                          = ( k + 1) z p.
                                       k

                                          1− m
                                      yn
    and            Q = ( k + 1) z q.
                                   k

                                     1− n
 Therefore the given equation reduces to
                       1 − m   1 − n  
                    f         P,     Q  = 0
                       k + 1   k + 1  
 This is of type 1 equation.


Problem:
           p2 q2          
        z2 2 + 2
(1)Solve                   =1
                           
          x   y           
Solution:

             2 p   q2 
                  2

     Given: z  2 + 2  = 1
              x
                   y 

      It can be rewritten as      (x   −1
                                            zp ) + ( y −1 zq ) = 1
                                               2            2
                                                                               ………..(1)

      which is of the form ( x m z k p ) + ( y n z k q ) = 1
                                              2             2



      we make the transformations
                               X = x 1− m , Y = y 1−n and Z = z k +1

                   i.e.        X = x 2 , Y = y 2 and Z = z 2
                               ∂z dz ∂Z dX  1
      Then                p=     =  .  .   = .P.2 x
                               ∂x dZ ∂X dx 2 z
                     ⇒ P = x −1 zp ,



                                                                                          23
Similarly,     Q = y −1 zq ,
       Using these in (1),it becomes
                    P2 + Q2 = 1                                 …………(2)
       As (2) contains only P and Q explicitly, a solution of the equation will be of the
       form
                   Z = aX + bY + c                             ………….(3)
       Therefore P = a and Q = b, obtained from (3) satisfy equation (2)
                    a 2 + b 2 = 1,
         i.e.
                    ⇒b = ± 1− a2
        Therefore the complete solution of (2) is

                     Z = aX ± 1 − a 2 Y + c
        Therefore the complete solution of (1) is

                     z 2 = ax 2 ± 1 − a 2 y 2 + c
         Singular solution does not exist. General solution is found out as usual.


Type D:
                        px qy 
Equation of the form f  ,  = 0
                        z z 
        By putting X = log x, Y = log y and Z = log z the equation reduces to

f ( P, Q ) = 0,
             ∂Z         ∂Z
where P =       and Q =    .
             ∂X         ∂Y


Problems:
(1)Solve pqxy = z 2 .
Solution:
    Given: pqxy = z 2 .                                               …………….(1)
     Rewriting (1),
                   px  qy 
                     = 1 .                                       …………….(2)
                   z  z 


                                                                                            24
 px    qy 
     As (2) contains   and   , we make the substitutions
                      z     z 
                  X = log x, Y = log y and Z = log z
                       ∂z dz ∂Z dX        1
    Then          P=     =  .  .   = z.P.
                       ∂x dZ ∂X dx        x
                   px
     i.e.             =P
                   z
                    qx
     Similarly,        =Q
                     z
     Using these in (2), it becomes
                     PQ = 1                                             …………..(3)
     which contains only P and Q explicitly. A solution of (3) is of the form
                    Z = aX + bY + c                                       …………(4)
     Therefore P = a and Q = b, obtained from (4) satisfy equation (3)
                                    1
       i.e.       ab = 1 or b =
                                    a
                                                       1
     Therefore the complete solution of (3) is Z = aX + Y + c
                                                       a
                                                                   1
     Therefore the complete solution of (1) is log z = a log x +     log y + c ……..(5)
                                                                   a
      General solution of (1) is obtained as usual.


General solution of partial differential equations:
        Partial differential equations, for which the general solution can be obtained
directly, can be divided in to the following three categories.
   (1) Equations that can be solved by direct (partial) integration.
   (2) Lagrange’s linear equation of the first order.
   (3) Linear partial differential equations of higher order with constant coefficients.


Equations that can be solved by direct (partial) integration:
Problems:




                                                                                           25
∂ 2u                                       ∂u
(1)Solve the equation             = e −t cos x, if u = 0 when t = 0 and    = 0 when x = 0.
                             ∂x∂t                                       ∂t
 Also show that u → sin x, when t → ∞ .
 Solution:
                ∂ 2u
      Given:         = e −t cos x,                                 …………….(1)
                ∂x∂t
      Integrating (1) partially with respect to x,
                 ∂u
                    = e −t sin x + f ( t )                         …………….(2)
                 ∂t
                            ∂u
      When t = 0 and           = 0 in (2), we get f ( t ) = 0 .
                            ∂t
                                        ∂u
      Equation (2) becomes                 = e −t sin x            …………….(3)
                                        ∂t
      Integrating (3) partially with respect to t, we get
                 u = −e −t sin x + g ( x )                          ……………(4)
      Using the given condition, namely u = 0 when t = 0 , we get
               0 = − sin x + g ( x ) or g ( x ) = sin x
      Using this value in (4), the required particular solution of (1) is
               u = sin x (1 − e − t )

      Now       t →∞
                                   {
                lim( u ) = sin x lim(1 − e −t )
                                    t →∞
                                                  }
                          = sin x
      i.e. when t → ∞, u → sin x .


                             ∂z               ∂z
(2) Solve the equation          = 3 x − y and    = − x + cos y simultaneously.
                             ∂x               ∂y
Solution: Given
                 ∂z
                    = 3x − y                                           ................(1)
                 ∂x
                 ∂z
                    = − x + cos y                                    ..................(2)
                 ∂y
 Integrating (1) partially with respect to x,



                                                                                             26
3x 2
                z=        − yx + f ( y )                                  …………..(3)
                      2
   Differentiating (1) partially with respect to y,
               ∂z
                  = − x + f ′( y )                                        …………...(4)
               ∂y
   Comparing (2) and (4), we get
                f ′( y ) = cos y
                f ( y ) = sin y + c                                       ...................(5)
   Therefore the required solution is
                     3x 2
               z=         − yx + sin y + c , where c is an arbitrary constant.
                      2


Lagrange’s linear equation of the first order:
        A linear partial differential equation of the first order , which is of the form
                     Pp + Qq = R

where P, Q, R are functions of x, y , z is called Lagrange’s linear equation.
working rule to solve Pp + Qq = R
(1)To solve Pp + Qq = R , we form the corresponding subsidiary simultaneous equations
  dx dy dz
    =   = .
  P   Q  R
(2)Solving these equations, we get two independent solutions u = a and v = b .
(3)Then the required general solution is f ( u , v ) = 0 or u = φ ( v ) or v = ψ ( u ) .
                                                           dx dy dz
Solution of the simultaneous equations                       =   = .
                                                           P   Q  R
Methods of grouping:
       By grouping any two of three ratios, it may be possible to get an ordinary
differential equation containing only two variables, eventhough P;Q;R are in
general, functions of x,y,z. By solving this equation, we can get a solution of the
simultaneous equations. By this method, we may be able to get two independent
solutions, by using different groupings.




                                                                                                   27
Methods of multipliers:
               If we can find a set of three quantities l,m,n which may be constants or
functions of the variables x,y,z, such that      lP + mQ + nR = 0 , then the solution of the
simultaneous equation is found out as follows.
                     dx dy dz ldx + mdy + ndz
                       =   =   =
                     P   Q   R   lP + mQ + nR
Since     lP + mQ + nR = 0, ldx + mdy + ndz = 0. If ldx + mdy + ndz = 0 is an exact

differential of some function u ( x, y , z ) , then we get du = 0. Integrating this, we get
                                     dx dy dz
u = a , which is a solution of         =   = .
                                     P   Q  R
Similarly, if we can find another set of independent multipliers l ′, m′, n ′, we can get
another independent solution v = b .


Problems:
(1)Solve xp + yq = x .
Solution:
     Given: xp + yq = x .
     This is of Lagrange’s type of PDE where P = x, Q = y, R = x .
                                           dx dy dz
        The subsidiary equations are          =   = .
                                            x   y  x
                                           dx dy
        Taking first two members              =
                                            x   y

        Integrating we get                log x = log y + log c1
                                           x
                                             = c1
                                           y
        i.e.                                                              ………….(1)
                                               x
                                           u=
                                               y
                                           dx dz
        Taking first and last members         =   .
                                            x   x
        i.e.                               dx = dz .
                                          x − z = c2
        Integrating we get                                                ………......(2)
                                          v = x−z


                                                                                               28
x        
         Therefore the solution of the given PDE is φ ( u , v ) = 0 i.e. φ  , x − z  = 0 .
                                                                           y        
                                                                                    
(2)Solve the equation ( x − 2 z ) p + ( 2 z − y ) q = y − x.
Solution:
     Given: ( x − 2 z ) p + ( 2 z − y ) q = y − x.
      This is of Lagrange’s type of PDE where P = x − 2 z , Q = 2 z − y, R = y − x .
                                                dx     dy    dz
       The subsidiary equations are                 =      =      .               ……….(1)
                                              x − 2z 2z − y y − x
                                                               dx + dy + dz
       Using the multipliers 1,1,1, each ratio in (1)=                      .
                                                                    0
                                   dx + dy + dz = 0 .
       Integrating, we get         x+ y+z =a                                ……………(2)
                                                                ydx + xdy + 2zdz
       Using the multipliers y,x,2z, each ratio in (1)=                          .
                                                                       0
                                   d ( xy ) + 2 zdz = 0 .

       Integrating, we get         xy + z 2 = b                             ……………(3)

       Therefore the general solution of the given equation is f ( x + y + z, xy + z 2 ) = 0 .


(3)Show that the integral surface of the PDE x( y 2 + z ) p − y ( x 2 + z ) q = ( x 2 − y 2 ) z .

Which contains the straight line x + y = 0, z = 1 is x 2 + y 2 + 2 xyz − 2 z + 2 = 0 .
Solution:
      The subsidiary equations of the given Lagrange ‘s equation are
                                 dx           dy          dz
                                       =             = 2            .
                             x( y + z ) − y ( x + z ) ( x − y 2 ) z
                                 2             2                                ……………(1)

                                                       1     1    1
                            1 1 1                        dx + dy + dz
      Using the multipliers , , , . each ratio in (1)= x     y    z .
                            x y z
                                                               0
                                    1     1    1
                                      dx + dy + dz = 0 .
                                    x     y    z
       Integrating, we get          xyz = a                                     ……………(2)




                                                                                                    29
xdx + ydy − dz
          Using the multipliers y,x,-1, each ratio in (1)=                                  .
                                                                                   0
                                         xdx + ydy − dz = 0 .

          Integrating, we get x 2 + y 2 − 2 z = b                                         ……………(3)
          The required surface has to pass through
                                        x+ y =0
                                                                                          ……………(4)
                                        z =1
          Using (4) in (2) and (3), we have
                                       − x2 = a
                                                                                          ……………(5)
                                       2x 2 − 2 = b
          Eliminating x in (5) we get,
                                        2a + b + 2 = 0                                   …………….(6)
          Substituting for a and b from (2) and (3) in (6), we get
                                  x 2 + y 2 + 2 xyz − 2 z + 2 = 0 , which is the equation of the required
          surface.


Linear P.D.E.S of higher order with constant coefficients:
                 The standard form of a homogeneous linear partial differential equation of the
 n th order with constant coefficients is
   ∂nz      ∂nz        ∂nz              ∂n z
a 0 n + a1 n −1 + a 2 n −2 2 + ... + a n n = R ( x, y )                                   …………….(1)
   ∂x     ∂x ∂y      ∂x ∂y              ∂y
where a’s are constants.
                                          ∂                         ∂
If we use the operators D ≡                     and       D′ ≡         , we can symbolically write equation (1)
                                          ∂x                        ∂y
as
     (a D
      0
            n
                + a1 D n −1 D ′ + ... + a n D ′ n ) z = R( x, y )                         …………….(2)

                                        f ( D, D ′) z = R ( x, y )                        …………….(3)
where f ( D, D ′) is a homogeneous polynomial of the n th degree in D and D ′ .
The method of solving (3) is similar to that of solving ordinary linear differential
equations with constant coefficients.




                                                                                                             30
The general solution of (3) is of the form z = (complementary function)+(particular
integral),where the complementary function is the R.H.S of the general solution of

                                                                                            1
f ( D, D ′) z = 0 and the particular integral is given symbolically by                             R ( x, y ) .
                                                                                       f ( D, D ′)

Complementary function of f ( D, D ′) z = R( x, y ) :
      C.F of the solution of f ( D, D ′) z = R( x, y ) is the R.H.S of the solution of
                    f ( D, D ′) z = 0 .                                                      …………(1)
      In this equation, we put D = m, D ′ = 1 ,then we get an equation which is called the
      auxiliary equation.
      Hence the auxiliary equation of (1) is
                     a 0 m n + a1 m n −1 + ... + a n = 0                                     ………….(2)

     Let the roots of this equation be             m1 , m2 ,...mn .

Case 1:
      The roots of (2) are real and distinct.
      The general solution is given by
                    z = φ1 ( y + m1 x ) + φ 2 ( y + m2 x ) + ... + φ n ( y + mn x )

Case 2:
      Two of the roots of (2) are equal and others are distinct.
      The general solution is given by
                    z = φ1 ( y + m1 x ) + φ 2 ( y + m1 x ) + ... + φ n ( y + mn x )

Case 3:
     ‘r’ of the roots of (2) are equal and others distinct.
       The general solution is given by
                    z = φ1 ( y + m1 x ) + xφ 2 ( y + m1 x ) + ... + x r −1φ r ( y + mr x )
To find particular integral:
Rule (1): If the R.H.S of a given PDE is f ( x, y ) = e ax +by , then
                                1
                  P.I =                 e ax +by
                           f ( D, D ′ )
             Put D = a, D ′ = b




                                                                                                                  31
1
                      P.I =               e ax +by           if f ( a, b ) ≠ 0
                               f ( a, b )

         If f ( a, b ) = 0, refer to Rule (4).
Rule (2): If the R.H.S of a given PDE is f ( x, y ) = sin ( ax + by ) or cos( ax + by ) , then
                                    1
                      P.I =                 sin ( ax + by ) or cos( ax + by )
                               f ( D, D ′ )

               Replace D 2 = −a 2 , D ′ 2 = −b 2 and DD ′ = −ab in f ( D, D ′) provided the
               denominator is not equal to zero.
              If the denominator is zero, refer to Rule (4).
Rule (3): If the R.H.S of a given PDE is f ( x, y ) = x m y n , then
                                    1
                      P.I =                 xm yn
                               f ( D, D ′ )
                          = { f ( D, D ′ ) }
                                               −1
                                                    (x   m
                                                             , yn )

             Expand { f ( D, D ′)} −1 by using Binomial Theorem and then operate on x m y n .

Rule (4): If the R.H.S of a given PDE f ( x, y ) is any other function [other than Rule(1),

(2) and(3)] resolve f ( D, D ′) into linear factors say ( D − m1 D ′)( D − m2 D ′)
etc. then the
                                         1
                 P.I =                                                f ( x, y )
                         ( D − m1 D ′)( D − m2 D ′)
 Note: If the denominator is zero in Rule (1) and (2) then apply Rule (4).
 Working rule to find P.I when denominator is zero in Rule (1) and Rule (2).
         If the R.H.S of a given PDE is of the form sin ( ax + by ) or cos( ax + by ) or e ax +by

                                    1                         xn
  Then                 P.I =                . f ( ax + by ) = n f ( ax + by )
                               ( bD − aD ′)                  b n!
  This rule can be applied only for equal roots.


Problems:
(1) Solve ( D 3 − 3DD ′ 2 + 2 D ′ 3 ) z = e 2 x − y + e x + y
 Solution:
      Given: ( D 3 − 3DD ′ 2 + 2 D ′ 3 ) z = e 2 x − y + e x + y



                                                                                                    32
The auxiliary equation is m 3 − 3m + 2 = 0
                                                  ⇒ m = 1,1,−2
               C.F = xf 1 ( y + x ) + f 2 ( y + x ) + f 3 ( y − 2 x )

       P.I =
                           1
                                          (e 2 x− y + e x+ y )
               D − 3D D ′ + 2 D ′
                  3          22        3

                        1                                       1
           =                          e 2 x− y +                            e x+ y
             ( D + 2 D ′)( D − D ′) 2
                                                   ( D − D ′) ( D + 2 D ′ )
                                                               2



             1    1               1      1
            = .         e 2 x− y + .             e x+ y
             9 D + 2D ′           9 ( D − D ′) 2


                1  2 x− y x 2 x+ y 
            =      xe    +   e 
                9          2       
The general solution of the given equation is
                                                                 x 2 x− y x 2 x+ y
         z = xf1 ( y + x ) + f 2 ( y + x ) + f 3 ( y − 2 x ) +     e     +    e
                                                                 9         18


(2)Solve ( D 2 + 4 DD ′ − 5 D ′ 2 ) z = xy + sin ( 2 x + 3 y )
  Solution:
      Given: ( D 2 + 4 DD ′ − 5 D ′ 2 ) z = xy + sin ( 2 x + 3 y )
      The auxiliary equation is m 2 + 4m − 5 = 0
                                                  ⇒ m = 1,−5
               C.F = φ1 ( y − 5 x ) + φ 2 ( y + x )
                                 1
                ( P.I ) 1 =                    ( xy )
                          D + 4 DD ′ − 5 D ′ 2
                                  2

                                 1
                      =                           ( xy )
                         2   D′                
                        D 1 + 2 ( 4 D − 5 D ′) 
                           D                   
                                                       −1
                        1    D′                
                      = 2 1 + 2 ( 4 D − 5 D ′)  ( xy )
                       D  D                    
                        1    D′                    
                      = 2 1 − 2 ( 4 D − 5D ′) + ...( xy )
                       D  D                        




                                                                                     33
1
                    =    2
                           ( xy ) − 14 .4 D ′( xy )
                      D             D
                        3
                      x y        1
                    =        − 4 ( 4x)
                        6      D
                      1 3          1 5
                    = x y−           x
                      6           30
                                   1
              ( P.I ) 2 =                        sin ( 2 x + 3 y )
                            D + 4 DD ′ − 5 D ′ 2
                              2


                                       1
                        =                                sin ( 2 x + 3 y )
                         ( − 2) + 4.( − 2.3) − 5( − 3) 2
                                   2


                          1
                        = sin ( 2 x + 3 y )
                         17
        Therefore the general solution is

                         z = φ1 ( y − 5 x ) + φ 2 ( y + x ) +
                                                                        6
                                                                          ( x y ) − 30 x 5 + 17 sin ( 2 x + 3 y )
                                                                        1 3          1        1




(3)Solve ( D 2 − 2 DD ′ + D ′ 2 ) z = x 2 y 2 e x + y
Solution:
       Given: ( D 2 − 2 DD ′ + D ′ 2 ) z = x 2 y 2 e x + y
       The auxiliary equation is m 2 − 2m + 1 = 0
                                                         ⇒ m = 1,1
              C.F = xf 1 ( y + x ) + f 2 ( y + x )

                                                             e x+ y ( x 2 y 2 )
                                                   1
                                  P.I =
                                           ( D − D ′)    2


                                                               1
                                       = e x+ y                               x2 y2
                                                   { ( D + 1) − ( D ′ + 1)} 2



                                                         1
                                       = e x+ y                     x2 y2
                                                  ( D − D ′)    2


                                                                            −2
                                                   1  D′ 
                                                      1 −  ( x y )
                                                  x+ y
                                         =e                      2 2

                                                  D2      D
                                                 1  2 D′      D′ 2 
                                       = e x + y 2 1 +     + 3 2 ( x 2 y 2 )
                                                D      D     D   




                                                                                                                    34
x y + ( 2 x y ) + 2 ( 2 x )
                                   1   2 2 2                     3     2 
                        = e x+ y                        2

                                   D2             D             D         

                    = e x + y  y 2 2 ( x 2 ) + 4 y. 3 ( x 2 ) + 6. 4 ( x 2 ) 
                                   1                1              1         
                               D                   D               D         
                      1               1           1 6  x+ y
                    =  x4 y 2 + x5 y +               x e
                       12            15           60 
Therefore the general solution is
                                                1      1    1 2  4 x+ y
            z = xf1 ( y + x ) + f 2 ( y + x ) +  y 2 + xy +   x x e
                                                 12   15    60 




                                                                                  35
UNIT 1
                                                 Part A
(1)Form a partial differential equation by eliminating arbitrary constants a and b from
   z = ( x + a ) + ( y + b)
                2           2


Ans:
    Given z = ( x + a ) 2 + ( y + b ) 2      ……. (1)
              ∂z
          p=      = 2( x + a )                 ( 2)
              ∂x
              ∂z
         q=       = 2( y + b )                 ( 3)
              ∂y
                                                 p2 q2
Substituting (2) & (3) in (1), we get z =          +
                                                 4   4
(2) Solve: ( D 2 − 2 DD ′ + D 2 ) z = 0
Ans:
  Auxiliary equation
           m 2 − 2m + 1 = 0
                  ( m − 1) 2 = 0
                          m = 1,1
                        z = f 1 ( y + x ) + xf 2 ( y + x )
(3)Form a partial differential equation by eliminating the arbitrary constants a and b from
the equation ( x − a ) 2 + ( y − b ) 2 = z 2 cot 2 α .
Ans:
        Given: ( x − a ) 2 + ( y − b ) 2 = z 2 cot 2 α          ….. (1)
   Partially differentiating with respect to ‘x’ and ‘y’ we get
                  2( x − a ) = 2 zp cot 2 α                     ….. (2)
                  2( y − b ) = 2 zq cot α  2
                                                                …... (3)
     (2) ⇒            x − a = zp cot 2 α                             …… (4)
     (3) ⇒           y − b = zq cot 2 α                               ….. (5)
  Substituting (4) and (5) in (1) we get
      z 2 p 2 cot 4 α + z 2 q 2 cot 4 α = z 2 cot 2 α .
     ⇒                        p 2 + q 2 = tan 2 α .
(4)Find the complete solution of the partial differential equation p 2 + q 2 − 4 pq = 0.
  Ans:
 Given:
                  p 2 + q 2 − 4 pq = 0.                   …….. (1)
  Let us assume that
                 z = ax + by + c                          ……… (2)


                                                                                           36
be the solution of (1)
  Partially differentiating with respect to ‘x’ and ‘y’ we get
                       ∂z
                 p=       =a
                       ∂x                          …….. (3)
                      ∂z
                 q=       =b
                      ∂y
  Substituting (3) in (1) we get
             a 2 + b 2 − 4ab = 0
  From the above equation we get,
               4b ± 16b 2 − 4a 2 b 2
          a=
                       2
          a = b ± b 4 − a2                     ………. (4)
  Substituting (5) in (2) we get
               (             )
          z = b 1 ± 4 − a 2 x + by + c
(5)Find the PDE of all planes having equal intercepts on the x and y axis.
Ans:
The equation of such plane is
     x y z
       + + =1                                      ………. (1)
     a a b
 Partially differentiating (1) with respect to ‘x’ and ‘y’ we get
           1 p                  b
             + =0⇒ p =−                           ……….. (2)
           a b                  a
           1 q                 b
             + =0⇒q =−                            ……….. (3)
           a b                 a
From (2) and (3), we get
                        p=q
(6)Find the solution of px 2 + qy 2 = z 2 .
Ans:
 The S.E is
             dx dy dz
                =     =
             x2 y2 z 2
Taking first two members, we get
              dx dy
                 =
              x2 y2
Integrating we get
            −1 −1
                 =   + c1
             x     y
               1 1
i.e          u  −  = c1
                y x
                    
Taking last two members, we get


                                                                             37
dy dz
                  =
                y2 z2
 Integrating we get
             −1 −1
                 =    + c2
              y     z
               1 1
i.e          v −  = c 2
               z y
                     
The complete solution is
         1 1 1 1
        φ − , −  = 0
          y x z y
                        
(7)Find the singular integral of the partial differential equation z = px + qy + p 2 − q 2 .
Ans:
The complete integral is
           z = ax + by + a 2 − b 2 .
       ∂z                            x
           = x + 2a = 0 ⇒ a = −
       ∂a                            2
       ∂z                        y
           = y − 2b = 0 ⇒ b =
       ∂b                        2

Therefore
             x2 y2 x2 y2         x2 y2
        z=−     +   +     −   =−   +
             2    2     4   4    4   4
    ⇒ y − x = 4z
         2    2


(8)Solve: 2
           p + q2 = m2.
Ans:
  Given     p2 + q2 = m2                                 …….. (1)
  Let us assume that
            z = ax + by + c                                …… (2)
  be the solution of (1)
  Partially differentiating with respect to ‘x’ and ‘y’ we get
                     ∂z
                 p=      =a
                     ∂x                                  …….. (3)
                     ∂z
                 q=      =b
                     ∂y
  Substituting (3) in (1) we get
             a2 + b2 = m2
  This is the required solution.



                                                                                               38
(9)Form a partial differential equation by eliminating the arbitrary constants a and b from
    z = ax n + by n .
Ans:
          Given z = ax n + by n .                       ……… (1)
  Partially differentiating with respect to ‘x’ and ‘y’ we get
                      ∂z                       p
                 p=      = a.nx n −1 ⇒ a = n −1
                      ∂x                    nx           …….. (2)
                     ∂z         n −1          q
                 q=      = b.ny      ⇒ b = n −1
                     ∂y                     ny
  Substituting (2) in (1) we get
                    p           q
             z = n −1 x n + n −1 y n
                  nx          ny
                1
            z = ( px + qy )
                n
 This is the required PDE.
(10)Solve:
             (D   3
                      + D 2 D ′ + DD ′ 2 + D ′ 3 ) z = 0.
Ans:
  Auxiliary equation
           m3 + m2 + m + 1 = 0
                      ( m + 1) 3 = 0
                             m = −1,−1,−1
                           z = f 1 ( y − x ) + xf 2 ( y − x ) + x 2 f 3 ( y − x )
(11)Form a partial differential equation by eliminating the arbitrary constants a and b
from
    z = ( x 2 + a 2 )( y 2 + b 2 ).
  Ans: Given z = ( x 2 + a 2 )( y 2 + b 2 )                                         ……. (1)
              ∂z
                 = 2 x( y 2 + b 2 ) ⇒ y 2 + b 2 =
                                                    p
        p=                                                                           ( 2 )
              ∂x                                   2x
              ∂z
                 = 2 y( x 2 + a 2 ) ⇒ x 2 + a 2 =
                                                    q
        q=                                                                           ( 3)
              ∂y                                  2y
                                                q p
Substituting (2) & (3) in (1), we get z =         .
                                               2 y 2x
                                         pq = 4 xyz
(12)Solve:
        ( D 2 − DD′ + D′ − 1) z = 0
Ans:
 The given equation can be written as


                                                                                               39
( D − 1)( D − D ′ + 1) z = 0
We know that the C.F corresponding to the factors
       ( D − m1 D ′ − α 1 )( D − m2 D′ − α 2 ) z = 0 is
          z = e α1x f 1 ( y + m1 x ) + e α 2 x f 2 ( y + m2 x )
In our problem
          α 1 = 1, α 2 = −1, m1 = 0, m2 = 1
       C.F = e x f 1 ( y + x ) + e − x f 2 ( y + x )
         z = e x f1 ( y + x ) + e − x f 2 ( y + x )
(13)Form a partial differential equation by eliminate the arbitrary function f from
           xy 
    z = f  .
           z 

Ans:
                xy 
Given : z = f  .
                z 
                xy  zy − xy. p
        p = f ′ ..                                              ............(1)
                z       z2
               xy  zx − xy.q
       q = f ′ ..                                           ............(2)
               z      z2

From (1), we get
         xy    pz 2
     f ′  =                                                .............(3)
         z  zy − xyp

Substituting (3) in(2), we get
                   pz 2    zy − xy. p
           p=            .
                zy − xyp      z2
(14)Solve:
              (D   3
                       + 2 D 2 D ′ − DD ′ 2 − 2 D ′ 3 ) z = 0.
Ans: Auxiliary equation
           m 3 + 2m 2 − m − 2 = 0
                     m = 1,−1,−2
           Solution is z = f 1 ( y + x ) + f 2 ( y − x ) + f 3 ( y − 2 x )
(15)Obtain partial differential equation by eliminating arbitrary constants a and b from
    ( x + a ) 2 + ( y + b) 2 + z 2 = 1.
Ans:
Given ( x − a ) 2 + ( y − b ) 2 + z 2 = 1                                           ……. (1)




                                                                                              40
2( x − a ) + 2 zp = 0 ⇒ ( x − a ) = − zp                 .........(2)
        2( y − b ) + 2 zq = 0 ⇒ ( y − b ) = − zq                  ( 3)
Substituting (2) & (3) in (1), we get
           z 2 p2 + z 2q2 + z 2 = 1
                                1
                 p2 + q2 +1 =
                                z2
(16)Find the general solution of
                  ∂2 z       ∂2 z    ∂2 z
                 4 2 − 12         + 9 2 = 0.
                  ∂y        ∂x∂y     ∂ y
Ans:
  Auxiliary equation is
                    4m 2 − 12m + 9 = 0
                12 ± 144 − 144
            m=
                        8
                3 3
           m= ,
                2 2
General solution is
                          3              3 
              z = f 1  y + x  + xf 2  y + x 
                          2              2 
(17)Find the complete integral of
                                               ∂z     ∂z
                 p + q = pq, where        p=      ,q = .
                                               ∂x     ∂y
 Ans: Let us assume that
               z = ax + by + c                     ……… (1)
  be the solution of the given equation.
  Partially differentiating with respect to ‘x’ and ‘y’ we get
                     ∂z
                 p=      =a
                     ∂x                            …….. (2)
                     ∂z
                 q=      =b
                     ∂y
  Substituting (2) in (1) we get
                                        a
                  a + b = ab ⇒ b =
                                      a −1
  Substituting the above in (1) we get
                            a 
                  z = ax +      y + c
                            a −1
  This gives the complete integral.
(18)Solve:



                                                                                41
(D   3
                     − 3 DD ′ 2 + 2 D ′ 3 ) z = 0
Ans:
  Auxiliary equation
             m 3 − 3m + 2 = 0
                          m = 1,1,−2
             Solution is z = f 1 ( y + x ) + xf 2 ( y + x ) + f 3 ( y − 2 x )
(19)Find the PDE of the family of spheres having their centers on the line x=y=z.
Ans: The equation of such sphere is
               ( x − a) 2 + ( y − a) 2 + ( z − a) 2 = r 2
Partially differentiating with respect to ‘x’ and ‘y’ we get
              2( x − a ) + 2( z − a ) p = 0                        ...........(1)
              2( y − a ) + 2( z − a ) q = 0                       ...........(2)
From (1),
                        x + zp
                a=                                                      ...........(3)
                        1+ p
From (2),
                         y + zq
                 a=                                                     ...........(4)
                         1+ q
From (3) and (4), we get
             x + zp y + zq
                    =
             1+ p      1+ q
This is the required PDE.
(20)Solve:
              ∂3 z    ∂2z   ∂3z      ∂3 z
                   −2 2 −4        + 8 3 = 0.
              ∂x 3   ∂x ∂y ∂x∂y 2    ∂y
Ans:
  Auxiliary equation
 m 3 − 2m 2 − 4 m + 8 = 0
           m = 2,2,−2
 Solution is z = f 1 ( y + 2 x ) + xf 2 ( y + 2 x ) + f 3 ( y − 2 x )

                                                      Part B
(1)(i) Form a partial differential equation by eliminating arbitrary functions from
z = xf ( 2 x + y ) + g ( 2 x + y )
(ii) Solve: p 2 y (1 + x 2 ) = qx 2

(2)(i) Solve: x( z 2 − y 2 ) p + y ( x 2 − z 2 ) q = z ( y 2 − x 2 )




                                                                                         42
∂3z     ∂3 z
(ii) Solve: 3  − 2 2 = e x + 2 y + 4 sin ( x + y )
           ∂x     ∂x ∂y

(3)(i) Solve: ( x 2 − y 2 − z 2 ) p + 2 xyq − 2 xz = 0
(ii) Solve: ( D 2 − DD ′ − 2 D ′ 2 ) z = 2 x + 3 y + e 3 x + 4 y

(4)(i) Solve: z 2 ( p 2 + q 2 ) = x 2 + y 2
(ii) Solve: ( D 2 + 3DD ′ − 4 D ′ 2 ) z = sin y

(5)(i) Solve: ( x 2 + y 2 + yz ) p + ( x 2 + y 2 − xz ) q = z ( x + y )
(ii) Solve: ( D 2 − DD ′ − 20 D ′ 2 ) z = e 5 x + y + sin ( 4 x − y )

(6)(i) Solve: z = p 2 + q 2
(ii) Solve: ( D 2 + DD ′ − 6 D ′ 2 ) z = x 2 y + e 3 x + y

(7)(i) Solve: ( y 2 + z 2 ) p − xyq + xz = 0
(ii) Solve: ( D 2 − 6 DD ′ + 5D ′ 2 ) z = xy + e x sinh y

(8)(i) Solve: p (1 − q 2 ) = q (1 − z )
(ii) Solve: ( D 2 − 4 DD ′ + 4 D ′ 2 ) z = xy + e 2 x + y

(9)(i) Solve: ( x 2 − yz ) p + ( y 2 − zx ) q = z 2 − xy
(ii) Solve: x( y − z ) p + y ( z − x ) q = z ( x − y )

(10)(i) Solve: p 2 + x 2 y 2 q 2 = x 2 z 2
(ii)Solve: ( D 2 − D ′ 2 − 3D + 3D ′) z = xy

(11)(i) Form the partial differential equation by eliminating f and φ from
 z = f ( y) + φ( x + y + z)
(ii) Solve: ( D ′ 2 − 2 DD ′) z = x 3 y + e 2 x

(12)(i) Find the complete integral of p + q = x + y
(ii) Solve: y 2 p − xyq = x( z − 2 y )

(13)(i) Solve: ( 3 z − 4 y ) p + ( 4 x − 2 z ) q = 2 y − 3 x
(ii) Solve: ( D 2 − 2 DD ′ − D ′ 2 + 3D + 3D ′ + 2 ) z = ( e 3 x + 2e −2 y )
                                                                               2


(iii) Solve: ( D 2 + 4 DD ′ − 5 D ′ 2 ) z = sin ( x − 2 y ) + e 2 x − y

(14)(i) Solve z 2 = 1 + p 2 + q 2
(ii) Solve: ( y − z ) p − ( 2 x + y ) q = 2 x + z


                                                                                   43
(15)(i) Form a partial differential equation by eliminating arbitrary functions f and g in
 z = x 2 f ( y ) + y 2 g ( x)
(ii) Solve: ( D 2 − DD ′ − 20 D ′ 2 ) z = sin ( 4 x − y ) + e 5 x + y

(16)(i) Form a partial differential equation by eliminating arbitrary functions f and g in
 z = f ( x3 + 2 y) + g( x3 − 2 y)
(ii) Solve: ( y − xy ) p + ( yz − x ) q = ( x + y )( x − y )

(17)(i) Find the singular solution of z = px + qy +              p2 + q2 +1
(ii) Solve: ( D 2 − DD ′ − 30 D ′ 2 ) z = xy + e 6 x + y

(18) (i) Solve: ( D 3 − 7 DD ′ 2 − 6 D ′ 3 ) z = sin ( x + 2 y ) + e 2 x + y
(ii) Find the singular integral of a partial differential equation z = px + qy + p 2 − q 2

(19)(i) Solve: ( 4 D 2 − 4 DD ′ + D ′ 2 ) z = 16 log( x + 2 y )
(ii) Form a partial differential equation by eliminating arbitrary functions ‘f’ from
 f ( z − xy, x 2 + y 2 ) = 0


(20)(i) Solve: ( D 2 + D ′ 2 ) z = sin 2 x sin 3 y + 2 sin 2 ( x + y )
(ii) Solve: p 2 + x 2 y 2 q 2 = x 2 z 2




                                                                                             44

More Related Content

What's hot

Vector calculus
Vector calculusVector calculus
Vector calculus
raghu ram
 
Second order homogeneous linear differential equations
Second order homogeneous linear differential equations Second order homogeneous linear differential equations
Second order homogeneous linear differential equations
Viraj Patel
 
the fourier series
the fourier seriesthe fourier series
the fourier series
safi al amu
 
Applications of Integrations
Applications of IntegrationsApplications of Integrations
Applications of Integrations
itutor
 
Ode powerpoint presentation1
Ode powerpoint presentation1Ode powerpoint presentation1
Ode powerpoint presentation1
Pokkarn Narkhede
 
Newton-Raphson Method
Newton-Raphson MethodNewton-Raphson Method
Newton-Raphson Method
Jigisha Dabhi
 
Ordinary differential equations
Ordinary differential equationsOrdinary differential equations
Ordinary differential equations
Ahmed Haider
 

What's hot (20)

Higher order ODE with applications
Higher order ODE with applicationsHigher order ODE with applications
Higher order ODE with applications
 
Differential equations
Differential equationsDifferential equations
Differential equations
 
Fourier series
Fourier seriesFourier series
Fourier series
 
presentation on Euler and Modified Euler method ,and Fitting of curve
presentation on Euler and Modified Euler method ,and Fitting of curve presentation on Euler and Modified Euler method ,and Fitting of curve
presentation on Euler and Modified Euler method ,and Fitting of curve
 
Vector calculus
Vector calculusVector calculus
Vector calculus
 
Green Theorem
Green TheoremGreen Theorem
Green Theorem
 
Higher Differential Equation
Higher Differential Equation Higher Differential Equation
Higher Differential Equation
 
Second order homogeneous linear differential equations
Second order homogeneous linear differential equations Second order homogeneous linear differential equations
Second order homogeneous linear differential equations
 
Multiple ppt
Multiple pptMultiple ppt
Multiple ppt
 
Calculus of variations
Calculus of variationsCalculus of variations
Calculus of variations
 
Cauchy integral theorem & formula (complex variable & numerical method )
Cauchy integral theorem & formula (complex variable & numerical method )Cauchy integral theorem & formula (complex variable & numerical method )
Cauchy integral theorem & formula (complex variable & numerical method )
 
the fourier series
the fourier seriesthe fourier series
the fourier series
 
Applications of Integrations
Applications of IntegrationsApplications of Integrations
Applications of Integrations
 
Ode powerpoint presentation1
Ode powerpoint presentation1Ode powerpoint presentation1
Ode powerpoint presentation1
 
Lesson 22: Quadratic Forms
Lesson 22: Quadratic FormsLesson 22: Quadratic Forms
Lesson 22: Quadratic Forms
 
Mech MA6351 tpde_notes
Mech MA6351 tpde_notes Mech MA6351 tpde_notes
Mech MA6351 tpde_notes
 
Introduction of Partial Differential Equations
Introduction of Partial Differential EquationsIntroduction of Partial Differential Equations
Introduction of Partial Differential Equations
 
Line integral,Strokes and Green Theorem
Line integral,Strokes and Green TheoremLine integral,Strokes and Green Theorem
Line integral,Strokes and Green Theorem
 
Newton-Raphson Method
Newton-Raphson MethodNewton-Raphson Method
Newton-Raphson Method
 
Ordinary differential equations
Ordinary differential equationsOrdinary differential equations
Ordinary differential equations
 

Similar to Chapter 1 (maths 3) (20)

MA8353 TPDE
MA8353 TPDEMA8353 TPDE
MA8353 TPDE
 
Quadratic functions and models
Quadratic functions and modelsQuadratic functions and models
Quadratic functions and models
 
Calculus Final Exam
Calculus Final ExamCalculus Final Exam
Calculus Final Exam
 
Engr 213 midterm 2b sol 2010
Engr 213 midterm 2b sol 2010Engr 213 midterm 2b sol 2010
Engr 213 midterm 2b sol 2010
 
Quadratic Function Presentation
Quadratic Function PresentationQuadratic Function Presentation
Quadratic Function Presentation
 
Advanced Engineering Mathematics Solutions Manual.pdf
Advanced Engineering Mathematics Solutions Manual.pdfAdvanced Engineering Mathematics Solutions Manual.pdf
Advanced Engineering Mathematics Solutions Manual.pdf
 
Quadric surfaces
Quadric surfacesQuadric surfaces
Quadric surfaces
 
Chapter 3
Chapter 3Chapter 3
Chapter 3
 
Engr 213 final sol 2009
Engr 213 final sol 2009Engr 213 final sol 2009
Engr 213 final sol 2009
 
maths
maths maths
maths
 
整卷
整卷整卷
整卷
 
Em05 pe
Em05 peEm05 pe
Em05 pe
 
Online math tutors (1)
Online math tutors (1)Online math tutors (1)
Online math tutors (1)
 
Quadratic equations
Quadratic equationsQuadratic equations
Quadratic equations
 
009 solid geometry
009 solid geometry009 solid geometry
009 solid geometry
 
Emat 213 midterm 2 fall 2005
Emat 213 midterm 2 fall 2005Emat 213 midterm 2 fall 2005
Emat 213 midterm 2 fall 2005
 
Engr 213 sample midterm 2b sol 2010
Engr 213 sample midterm 2b sol 2010Engr 213 sample midterm 2b sol 2010
Engr 213 sample midterm 2b sol 2010
 
Maths assignment
Maths assignmentMaths assignment
Maths assignment
 
First Quarter - Chapter 2 - Quadratic Equation
First Quarter - Chapter 2 - Quadratic EquationFirst Quarter - Chapter 2 - Quadratic Equation
First Quarter - Chapter 2 - Quadratic Equation
 
Chapter 1 (math 1)
Chapter 1 (math 1)Chapter 1 (math 1)
Chapter 1 (math 1)
 

Recently uploaded

Artificial Intelligence: Facts and Myths
Artificial Intelligence: Facts and MythsArtificial Intelligence: Facts and Myths
Artificial Intelligence: Facts and Myths
Joaquim Jorge
 

Recently uploaded (20)

presentation ICT roal in 21st century education
presentation ICT roal in 21st century educationpresentation ICT roal in 21st century education
presentation ICT roal in 21st century education
 
Partners Life - Insurer Innovation Award 2024
Partners Life - Insurer Innovation Award 2024Partners Life - Insurer Innovation Award 2024
Partners Life - Insurer Innovation Award 2024
 
08448380779 Call Girls In Friends Colony Women Seeking Men
08448380779 Call Girls In Friends Colony Women Seeking Men08448380779 Call Girls In Friends Colony Women Seeking Men
08448380779 Call Girls In Friends Colony Women Seeking Men
 
A Domino Admins Adventures (Engage 2024)
A Domino Admins Adventures (Engage 2024)A Domino Admins Adventures (Engage 2024)
A Domino Admins Adventures (Engage 2024)
 
08448380779 Call Girls In Civil Lines Women Seeking Men
08448380779 Call Girls In Civil Lines Women Seeking Men08448380779 Call Girls In Civil Lines Women Seeking Men
08448380779 Call Girls In Civil Lines Women Seeking Men
 
Boost PC performance: How more available memory can improve productivity
Boost PC performance: How more available memory can improve productivityBoost PC performance: How more available memory can improve productivity
Boost PC performance: How more available memory can improve productivity
 
Driving Behavioral Change for Information Management through Data-Driven Gree...
Driving Behavioral Change for Information Management through Data-Driven Gree...Driving Behavioral Change for Information Management through Data-Driven Gree...
Driving Behavioral Change for Information Management through Data-Driven Gree...
 
Strategies for Unlocking Knowledge Management in Microsoft 365 in the Copilot...
Strategies for Unlocking Knowledge Management in Microsoft 365 in the Copilot...Strategies for Unlocking Knowledge Management in Microsoft 365 in the Copilot...
Strategies for Unlocking Knowledge Management in Microsoft 365 in the Copilot...
 
Mastering MySQL Database Architecture: Deep Dive into MySQL Shell and MySQL R...
Mastering MySQL Database Architecture: Deep Dive into MySQL Shell and MySQL R...Mastering MySQL Database Architecture: Deep Dive into MySQL Shell and MySQL R...
Mastering MySQL Database Architecture: Deep Dive into MySQL Shell and MySQL R...
 
[2024]Digital Global Overview Report 2024 Meltwater.pdf
[2024]Digital Global Overview Report 2024 Meltwater.pdf[2024]Digital Global Overview Report 2024 Meltwater.pdf
[2024]Digital Global Overview Report 2024 Meltwater.pdf
 
How to Troubleshoot Apps for the Modern Connected Worker
How to Troubleshoot Apps for the Modern Connected WorkerHow to Troubleshoot Apps for the Modern Connected Worker
How to Troubleshoot Apps for the Modern Connected Worker
 
The Role of Taxonomy and Ontology in Semantic Layers - Heather Hedden.pdf
The Role of Taxonomy and Ontology in Semantic Layers - Heather Hedden.pdfThe Role of Taxonomy and Ontology in Semantic Layers - Heather Hedden.pdf
The Role of Taxonomy and Ontology in Semantic Layers - Heather Hedden.pdf
 
How to convert PDF to text with Nanonets
How to convert PDF to text with NanonetsHow to convert PDF to text with Nanonets
How to convert PDF to text with Nanonets
 
Automating Google Workspace (GWS) & more with Apps Script
Automating Google Workspace (GWS) & more with Apps ScriptAutomating Google Workspace (GWS) & more with Apps Script
Automating Google Workspace (GWS) & more with Apps Script
 
From Event to Action: Accelerate Your Decision Making with Real-Time Automation
From Event to Action: Accelerate Your Decision Making with Real-Time AutomationFrom Event to Action: Accelerate Your Decision Making with Real-Time Automation
From Event to Action: Accelerate Your Decision Making with Real-Time Automation
 
Evaluating the top large language models.pdf
Evaluating the top large language models.pdfEvaluating the top large language models.pdf
Evaluating the top large language models.pdf
 
Artificial Intelligence: Facts and Myths
Artificial Intelligence: Facts and MythsArtificial Intelligence: Facts and Myths
Artificial Intelligence: Facts and Myths
 
Scaling API-first – The story of a global engineering organization
Scaling API-first – The story of a global engineering organizationScaling API-first – The story of a global engineering organization
Scaling API-first – The story of a global engineering organization
 
Strategize a Smooth Tenant-to-tenant Migration and Copilot Takeoff
Strategize a Smooth Tenant-to-tenant Migration and Copilot TakeoffStrategize a Smooth Tenant-to-tenant Migration and Copilot Takeoff
Strategize a Smooth Tenant-to-tenant Migration and Copilot Takeoff
 
Tech Trends Report 2024 Future Today Institute.pdf
Tech Trends Report 2024 Future Today Institute.pdfTech Trends Report 2024 Future Today Institute.pdf
Tech Trends Report 2024 Future Today Institute.pdf
 

Chapter 1 (maths 3)

  • 1. CHAPTER 1 PARTIAL DIFFERENTIAL EQUATIONS A partial differential equation is an equation involving a function of two or more variables and some of its partial derivatives. Therefore a partial differential equation contains one dependent variable and one independent variable. Here z will be taken as the dependent variable and x and y the independent variable so that z = f ( x, y ) . We will use the following standard notations to denote the partial derivatives. ∂z ∂z ∂2z ∂2z ∂2z = p, = q, 2 = r , = s, 2 = t ∂x ∂y ∂x ∂x∂y ∂y The order of partial differential equation is that of the highest order derivative occurring in it. Formation of partial differential equation: There are two methods to form a partial differential equation. (i) By elimination of arbitrary constants. (ii) By elimination of arbitrary functions. Problems Formation of partial differential equation by elimination of arbitrary constants: (1)Form the partial differential equation by eliminating the arbitrary constants from z = ax + by + a 2 + b 2 . Solution: 1
  • 2. Given z = ax + by + a 2 + b 2 ……………... (1) Here we have two arbitrary constants a & b. Differentiating equation (1) partially with respect to x and y respectively we get ∂z =a⇒ p=a ……………… (2) ∂x ∂z =b⇒q=a ………………. (3) ∂y Substitute (2) and (3) in (1) we get z = px + qy + p 2 + q 2 , which is the required partial differential equation. (2) Form the partial differential equation by eliminating the arbitrary constants x2 y2 z 2 a, b, c from + + = 1. a2 b2 c2 Solution: We note that the number of constants is more than the number of independent variable. Hence the order of the resulting equation will be more than 1. x2 y2 z 2 + + =1 .................. (1) a2 b2 c2 Differentiating (1) partially with respect to x and then with respect to y, we get 2x 2 y + p=0 .....................(2) a2 c2 2 y 2z + q=0 ......................(3) b2 c2 Differentiating (2) partially with respect to x, 1 1 2 + 2 ( zr + p 2 ) ……………..(4) a c ∂2z Where r = 2 . ∂x c 2 zp − 2 = ......................(5) a x From (2) and (4) , 2 . c − 2 = zr + p 2 ......................(6) a 2
  • 3. From (5) and (6), we get 2 ∂2z  ∂z  ∂z xz 2 + x  = z , which is the required partial differential ∂x  ∂x  ∂x equation. (3) Find the differential equation of all spheres of the same radius c having their center on the yoz-plane. . Solution: The equation of a sphere having its centre at ( 0, a, b ) , that lies on the yoz -plane and having its radius equal to c is x 2 + ( y − a ) 2 + ( z − b) 2 = c 2 ……………. (1) If a and b are treated as arbitrary constants, (1) represents the family of spheres having the given property. Differentiating (1) partially with respect to x and then with respect to y, we have 2 x + 2( z − b ) p = 0 …………… (2) and 2( y − a ) + 2( z − b ) q = 0 …………….(3) x From (2), z −b = − …………….(4) p qx Using (4) in (3), y − b = ……………..(5) p Using (4) and (5) in (1), we get q2x2 x2 x + 2 + 2 = c2 2 p p . i.e. (1 + p 2 + q 2 ) x 2 = c 2 p 2 , which is the required partial differential equation. Problems 3
  • 4. Formation of partial differential equation by elimination of arbitrary functions: (1)Form the partial differential equation by eliminating the arbitrary function ‘f’ from z = e ay f ( x + by ) solution: Given z = e ay f ( x + by ) i.e. e − ay z = f ( x + by ) ……………(1) Differentiating (1) partially with respect to x and then with respect to y, we get e − ay p = f ' ( u ).1 ………….(2) e − ay q − ae − ay z = f ' ( u ).b …………….(3) where u = x + by Eliminating f’(u) from (2) and (3), we get q − az =b p i.e. q = az + bp (2) Form the partial differential equation by eliminating the arbitrary function ‘ φ ’  x φ  z 2 − xy,  = 0  z  x Solution: Given φ  z 2 − xy,  = 0 ………………(1)  z x Let u = z 2 − xy , v= z Then the given equation is of the form φ ( u , v ) = 0 . The elimination of φ from equation (2), we get, ∂u ∂v ∂x ∂x = 0 ∂u ∂v ∂y ∂y 4
  • 5. z − px 2 zp − y i.e. z2 = 0 − xq 2 zq − x z2 i.e ( 2 zp − y )  − xq  −  z − 2px ( 2 zq − x ) = 0  2     z   z  i.e px 2 − q ( xy − 2 z 2 ) = zx (3) Form the partial differential equation by eliminating the arbitrary function ‘f’ from z = f ( 2 x + y ) + g ( 3x − y ) Solution: Given z = f ( 2 x + y ) + g ( 3x − y ) .…………(1) Differentiating (1) partially with respect to x, p = f ′( u ).2 + g ′( v ).3 ………….(2) Where u = 2 x + y and v = 3 x − y Differentiating (1) partially with respect to y, q = f ′( u ).1 + g ′( v ) (−1) …………. (3) Differentiating (2) partially with respect to x and then with respect to y, r = f ′′( u ).4 + g ′′( v ).9 …………. (4) and s = f ′′( u ).2 + g ′′( v ).(−3) ………….. (5) Differentiating (3) partially with respect to y, t = f ′′( u ).1 + g ′′( v ).1 ………….. (6) Eliminating f ′′( u ) and g ′′( v ) from (4), (5) and (6) using determinants, we have 4 9 r 2 −3 s = 0 1 1 t i.e. 5r + 5s − 30t = 0 ∂2z ∂2z ∂2z or + −6 2 = 0 ∂x 2 ∂x∂y ∂y 5
  • 6. (4) Form the partial differential equation by eliminating the arbitrary function ‘ φ ’ from 1 z = φ ( y − x ) + φ ′( y − x ) . x 1 Solution: Given z = φ ( u ) + φ ′( u ) …………...(1) x Where u = y − x Differentiating partially with respect to x and y, we get 1 1 p = φ ′( u ) (−1) − 2 φ ( u ) + φ ′′( u ) (−1) …………(2) x x 1 q = φ ′( u ).1 + φ ′′( u ).1 ………….(3) x 1 2 2 r= φ ′′( u ).1 + 2 φ ′( u ) + 3 φ ( u ) + φ ′′′( u ).1 ………..(4) x x x 1 1 s=− 2 φ ′( u ) − φ ′′( u ) + φ ′′′( u ) (−1) …………(5) x x 1 t = φ ′′( u ).1 + φ ′′′( u ).1 …………..(6) x From (4) and (6), we get 2 2 r −t = 2 φ ′( u ) + 3 φ ( u ) x x 2 1  =  φ ( u ) + φ ′( u )  x2 x  2 = z x2  ∂ 2 z ∂ 2z  i.e. x2  2 − 2  ∂x  = 2z   ∂y  Solutions of partial differential equations Consider the following two equations z = ax + by ………..(1) 6
  • 7.  y and z = xf   ………..(2) x Equation (1) contains arbitrary constants a and b, but equation (2) contains only one arbitrary function f. If we eliminate the arbitrary constants a and b from (1) we get a partial differential equation of the form xp + yq = z . If we eliminate the arbitrary function f from (2) we get a partial differential equation of the form xp + yq = z . Therefore for a given partial differential equation we may have more than one type of solutions. Types of solutions: (a) A solution in which the number of arbitrary constants is equal to the number of independent variables is called Complete Integral (or) Complete solution. (b) In complete integral if we give particular values to the arbitrary constants we get Particular Integral. (c) The equation which does not have any arbitrary constants is known as Singular Integral. To find the general integral: Suppose that f ( x, y, z, p, q ) = 0 ………....(1) is a first order partial differential equation whose complete solution is φ ( x, y , z , a , b ) = 0 ………..(2) Where a and b are arbitrary constants. Let b = f ( a ) , where ‘f’is an arbitrary function. Then (2) becomes φ ( x, y , z , a , f ( a ) ) = 0 ……….(3) Differentiating (3) partially with respect to ‘a’, we get ∂φ ∂φ + . f ′( a ) = 0 ……….(4) ∂a ∂b 7
  • 8. The eliminant of ‘a’ between the two equations (3) and (4), when it exists, is called the general integral of (1). Methods to solve the first order partial differential equation: Type 1: Equation of the form f ( p, q ) = 0 ………...(1) i.e the equation contains p and q only. Suppose that z = ax + by + c …….....(2) is a solution of the equation ∂z ∂z = a, = b ∂x ∂y ⇒ p = a, q = b substitute the above in (1), we get f ( a, b ) = 0 on solving this we can get b = φ ( a ) , where φ is a known function. Using this value of b in (2), the complete solution of the given partial differential equation is z = ax + φ ( a ) y + c …………(3) is a complete solution, To find the singular solution, we have to eliminate ‘a’ and ‘c’ from z = ax + φ ( a ) y + c Differentiating the above with respect to ‘a’ and ‘c’, we get 0 = x + φ ′( a ) y , and 0=1. The last equation is absurd. Hence there is no singular solution for the equation of Type 1. Problems: (1) Solve p2 + q2 = 1. 8
  • 9. Solution: Given: p 2 + q 2 = 1 ………….(1) Equation (1) is of the form f ( p, q ) = 0 . Assume z = ax + by + c ………….(2) be the solution of equation (1). From (2) we get p = a, q = b . (1) ⇒ a 2 + b 2 = 1 ⇒ b = ± 1− a2 ……….(3) Substitute (3) in (2) we get z = ax ± 1 − a 2 y + c ……....(4) This is a complete solution. To find the general solution: We put c = f ( a ) in (4),where ‘f’ is an arbitrary function. i.e. z = ax ± 1 − a 2 y + f ( a ) …………(5) Differentiating (5) partially with respect to ‘a’, we get a x y + f ′( a ) = 0 ……………(6) 1− a2 Eliminating ‘a’ between equations (5) and (6), we get the required general solution. To find the singular solution: Differentiate (4) partially with respect to ‘a’ and ‘c’, we get a 0= x± , 1− a2 0=1.(which is absurd) so there is no singular solution. (2) Solve p + q = pq 9
  • 10. Solution: Given: p + q = pq ………..(1) Equation (1) is of the form f ( p, q ) = 0 Assume z = ax + by + c ………(2) be the solution of equation (1). From (2) we get p = a, q = b (1) ⇒ a + b = ab a ⇒b= …….....(3) a −1 Substituting (3) in (2), we get a z = ax + y+c ………(4) a −1 This is a complete solution. To find the general solution: We put c = f ( a ) in (4), we get a z = ax + y + f ( a) ……..(5) a −1 Differentiating (5) partially with respect to ‘a’, we get 1 x− y + f ′( a ) ………..(6) ( a − 1) 2 Eliminating ‘a’ between equations (5) and (6), we get the required general solution To find the singular solution: Differentiating (4) with respect to ‘a’ and ‘c’. 1 0= x− y, ( a − 1) 2 and 0=1 (which is absurd). So there is no singular solution. 10
  • 11. Type 2: (Clairaut’s type) The equation of the form z = px + qy + f ( p, q ) ………(1) is known as Clairaut’s equation. Assume z = ax + by + c …………(2) be a solution of (1). ∂z ∂z = a, = b ∂x ∂y ⇒ p = a, q = b Substitute the above in (1), we get z = ax + by + f (a, b) ………..(3) which is the complete solution. Problem: (1) Solve z = px + qy + pq Solution: Given: z = px + qy + pq ……….(1) Equation (1) is a Clairaut’s equation Let z = ax + by + c ………..(2) be the solution of (1). Put p = a, q = b in (1), we get z = ax + by + ab ……….(3) which is a complete solution. To find the general solution: We put b = f ( a ) in (3), we get z = ax + f ( a ) y + af ( a ) ………(4) Differentiate (4) partially with respect to ‘a’, we get 11
  • 12. 1 0 = x + f ′( a ) y + [ af ′( a ) + f ( a ) ] ………..(5) 2 af ( a ) Eliminating ‘a’ between equations (4) and (5), we get the required general solution To find singular solution, Differentiate (3) partially with respect to ‘a’, we get 1 0= x+ .b 2 ab b ⇒x=− ………..(6) 2 a Differentiate (3) partially with respect to ‘b’, we get 1 0= y+ .a 2 ab a ⇒ y=− ………..(7) 2 b Multiplying equation (6) and (7),we get  b  a  xy =  −    2 a  − 2 b     1 xy = 4 4 xy = 1 q (2) Solve z = px + qy + −p p Solution: q Given: z = px + qy + −p ……….(1) p Equation (1) is a Clairaut’s equation Let z = ax + by + c ………...(2) be the solution of (1). Put p = a, q = b in (1), we get b z = ax + by + −a ……….(3) a 12
  • 13. which is the complete solution. To find the general solution: We put b = f ( a ) in (3), we get f ( a) z = ax + f ( a ) y + −a ……..(4) a Differentiate (4) partially with respect to ‘a’, we get af ′( a ) − f ( a ) 0 = x + f ′( a ) y + −1 ……..(5) a Eliminating ‘a’ between equations (4) and (5), we get the required general solution To find the singular solution: Differentiate (3) partially with respect to ‘a’, b 0= x− −1 a2 b ⇒x= +1 a2 ⇒ a2x = b +1 ⇒ b = a2 x −1 .............. (4) Differentiate (3) partially with respect to ‘b’, 1 0= y+ a 1 ⇒a=− ………….(5) y Substituting equation (4) and (5) in equation (3), we get  −1 z =   x + ( a 2 x − 1) y + ( a 2 x − 1) −  − 1     y  −1  y      y −x 1 z= + a 2 yx − y − a 2 xy + y + y y 13
  • 14. −x 1 z= + y y yz = 1 − x Type 3: Equations not containing x and y explicitly, i.e. equations of the form f ( z , p, q ) = 0 ……….(1) For equations of this type ,it is known that a solution will be of the form z = φ ( x + ay ) ……….(2) Where ‘a’ is the arbitrary constant and φ is a specific function to be found out. Putting x + ay = u , (2) becomes z = φ ( u ) or z ( u ) dz ∂u dz ∴ p= . = du ∂x du dz ∂u dz and q= . =a du ∂y du If (2) is to be a solution of (1), the values of p and q obtained should satisfy (1).  dz dz  i.e. f  z, , a  = 0 ……..(3)  du du  From (3), we get dz = ψ ( z, a ) ……….(4) du Now (4) is a ordinary differential equation, which can be solved by variable separable method. The solution of (4), which will be of the form g ( z, a ) = u + b or g ( z, a ) x + ay + b , is the complete solution of (1). The general and singular solution of (1) can be found out by usual method. Problems: (1)Solve z = p 2 + q 2 . Solution: 14
  • 15. Given: z = p 2 + q 2 …………(1) Equation (1) is of the form f ( z , p, q ) = 0 Assume z = φ ( u ) where, u = x + ay be a solution of (1). ∂z dz ∂u dz dz p= = . = ⇒ p= …….(2) ∂x du ∂x du du ∂z dz ∂u dz dz q= = . =a ⇒q=a ……(3) ∂y du ∂y du du Substituting equation (2) & (3) in (1), we get 2 2  dz   dz  z =   + a2   du   du  2 z =   (1 + a 2 )  dz   du  2  dz  z   =  du  1+ a2 1 dz z2 = du (1 + a 2 ) By variable separable method, dz du = (1 + a ) 1 1 2 2 z 2 By integrating, we get dz 1 ∫ = ∫ du + c (1 + a ) 1 1 2 3 z 2 2 z= ( x + ay ) +c (1 + a ) 1 2 2 z= ( x + ay ) + c 2(1 + a ) 1 2 2 2 z= ( x + ay ) +k ……….(4) 2(1 + a ) 1 2 2 15
  • 16. This is the complete solution. To find the general solution: We put k = f ( a ) in (4), we get z= ( x + ay ) + f ( a) ……..(5) 2(1 + a ) 1 2 2 Differentiate (5) partially with respect to ‘a’, we get y − xa 0= + f ′( a ) ………..(6) 2(1 + a ) 3 2 2 Eliminating ‘a’ between equations (4) and (5), we get the required general solution. To find the singular solution: Differentiate (4) partially with respect to ‘a’ and ‘k’, we get y − xa 0= ………..(7) 2(1 + a ) 3 2 2 and 0 = 1 (which is absurd) So there is no singular solution. (2)Solve 9( p 2 z + q 2 ) = 4 . Solution: Given: 9( p 2 z + q 2 ) = 4 ………(1) Equation (1) is of the form f ( z , p, q ) = 0 Assume z = φ ( u ) where , u = x + ay be a solution of (1). ∂z dz ∂u dz dz p= = . = ⇒ p= …….(2) ∂x du ∂x du du ∂z dz ∂u dz dz q= = . =a ⇒q=a ……(3) ∂y du ∂y du du 16
  • 17. Substituting equation (2) & (3) in (1), we get   dz  2 2 2  dz    9   z + a   = 4   du   du     2  dz  [ 9  z + a 2 = 4 ]  du  2  dz  4   =  du  9 z + a2 [ ] dz 2 1 =± . du 3 z + a2 2 z + a 2 dz = ± du 3 Integrating the above, we get ∫ (z + a ) 1 2 3∫ 2 2 dz = ± du + c (z + a2 ) 2 = ± 2 u + c 3 2 3 3 ( z + a 2 ) 2 = ± 2 ( x + ay ) + c 3 2 3 3 ⇒ ( z + a 2 ) 2 = ±( x + ay ) + k 3 ………..(4) This is the complete solution. To find the general solution: We put k = f ( a ) in (4), we get (z + a ) 3 2 2 = ±( x + ay ) + f ( a ) ……..(5) Differentiate (5) partially with respect to ‘a’, we get − 3a ( z − a 2 ) 2 = ± y + f ′( a ) 1 ……..(6) Eliminating ‘a’ between equations (4) and (5), we get the required general solution. To find the singular solution: Differentiate (4) partially with respect to ‘a’ and ‘k’, we get 17
  • 18. 3a ( z + a 2 ) 2 = ± y 1 ………..(7) and 0 = 1 (which is absurd) So there is no singular solution. Type 4: Equations of the form f ( x, p ) = g ( y , q ) ………..(1) i.e. equation which do not contain z explicitly and in which terms containing p and x can be separated from those containing q and y. To find the complete solution of (1), We assume that f ( x, p ) = g ( y , q ) = a .where ‘a’ is an arbitrary constant. Solving f ( x, p ) = a ,we can get p = φ ( x, a ) and solving g ( y , q ) = a ,we can get q = ψ ( y, a ) . Now ∂z ∂z dz = dx + dy or pdx + qdy ∂x ∂y i.e. dz = φ ( x, a ) dx + ψ ( y, a ) dy Integrating with respect to the concerned variables, we get z = ∫ φ ( x, a )dx + ∫ψ ( y, a ) dy + b ……….(2) The complete solution of (1) is given by (2), which contains two arbitrary constants ‘a’ and ‘b’. The general and singular solution of (1) can be found out by usual method. Problems: (1)Solve pq = xy . Solution: Given: pq = xy p y ⇒ = ………..(1) x q Equation (1) is of the form f ( x, p ) = g ( y , q ) 18
  • 19. p y Let = = a (say) x q p =a ⇒ p = ax ………….(2) x y y Similarly, =a ⇒q= …………(3) q a Assume dz = pdx + qdy be a solution of (1) Substitute equation (2) and (3) to the above, we get y dz = axdx + dy a Integrating the above we get, 1 ∫ dz = a ∫ xdx + a ∫ ydy + c ax 2 y 2 z= + +c 2 2a y2 2 z = ax +2 +k ………..(4) a This is the complete solution. The general and singular solution of (1) can be found out by usual method. (2) Solve p + q = x + y . Solution: Given: p + q = x + y ⇒p−x =q− y ………….. (1) Equation (1) is of the form f ( x, p ) = g ( y , q ) Let p − x = y − q = a (say) p−x =a ⇒ p = x+a …………. (2) Similarly, y − q = a ⇒ q = y−a ……………(3) Assume dz = pdx + qdy be a solution of (1) Substitute equation (2) and (3) to the above, we get dz = ( x + a ) dx + ( y − a ) dy Integrating the above we get, 19
  • 20. ∫ dz = ∫ ( x + a ) dx + ∫ ( y − a ) dy + c z= (x 2 + y2 ) + a( x − y ) + c 2 2 z = x 2 + y 2 + 2a ( x − y ) + c ……………(4) This is the complete solution. The general and singular solution of (1) can be found out by usual method. Equations reducible to standard types-transformations: Type A: Equations of the form f ( x m p, y n q ) = 0 or f ( x m p, y n q, z ) = 0 . Where m and n are constants, each not equal to 1. We make the transformations x 1− m = X and y 1− n = Y . ∂z ∂z ∂X ∂z Then p= = . = (1 − m ) x − m P, where P ≡ and ∂x ∂X ∂x ∂X ∂z ∂z ∂Y ∂z q= = . = (1 − n ) y −n Q, where Q ≡ ∂y ∂Y ∂y ∂Y Therefore the equation f ( x m p, y n q ) = 0 reduces to f { (1 − m ) P, (1 − n ) Q} = 0, .which is a type 1 equation. The equation f ( x m p, y n q, z ) = 0 reduces to f { (1 − m ) P, (1 − n ) Q, z} = 0, .which is a type 3 equation. Problem: (1)Solve p 2 x 4 + y 2 zq = 2z 2 . Solution: Given: p 2 x 4 + y 2 zq = 2z 2 This can be written as ( px ) + ( qy ) z = 2z . 2 2 2 2 Which is of the form f ( x p, y q, z ) = 0 , where m=2,n=2. m n 20
  • 21. 1− m 1 1 Put X = x = ; Y = y 1− n = x y ∂z ∂z ∂x P= = . = p ( − x 2 ) = − px 2 ∂X ∂x ∂X ∂z ∂z ∂y Q= = . = q ( − y 2 ) = −qy 2 ∂Y ∂y ∂Y Substituting in the given equation, P 2 − qz = 2z 2 . This is of the form f ( p, q, z ) = 0 . Let Z = f ( X + aY ) , where u = X + aY dz dz P= ,Q=a du du Equation becomes, . 2  dz  dz   − az − 2z 2 = 0  du  du dz dz az ± a 2 z 2 + 8 z 2 Solving for , = du du 2 dz a ± a 2 + 8 = du z 2 a ± a2 + 8 log z = ( X + aY ) + b 2 a ± a2 + 8  1 a  log z =  +  + b is a complete solution. x y 2   The general and singular solution can be found out by usual method. Type B: Equations of the form f ( z k p, z k q ) = 0 or f ( z k p , z k q, x, y ) = 0 . Where k is a constant, which is not equal to -1. We make the transformations Z = z k +1 . ∂Z Then P= = ( k + 1) z k p and ∂x 21
  • 22. ∂Z Q= = ( k + 1) z k p ∂y  P Q  Therefore the equation f ( z k p, z k q ) = 0 reduces to f  ,  = 0, which is a type 1  k +1 k + 1 equation.  P  f ( z k p, z k q, x, y ) = 0 reduces to f  Q The equation , , x, y  = 0, which is a  k +1 k +1  type 4 equation. Problems: (1)Solve: z 4 q 2 − z 2 p = 1. Solution: Given: z 4 q 2 − z 2 p = 1. The equation can be rewritten as ( z 2 q ) − ( z 2 p ) = 1 2 ………(1) Which contains z 2 p and z 2 q . Hence we make the transformation Z = z 3 ∂Z P= = 3z 2 p ∂x P ⇒ z2 p = 3 Q Similarly z q = 2 3 Using these values in (1), we get Q 2 − 3P = 9 ………..(2) As (2) is an equation containing P and Q only, a solution of (2) will be of the form Z = ax + by + c ………….(3) Now P = a and Q = b, obtained from (3) satisfy equation (2) b 2 − 3a = 9 i.e. b = ± 3a + 9 Therefore the complete solution of (2) is Z = ax ± 3a + 9 y + c i.e complete solution of (1) is z 3 = ax ± 3a + 9 y + c 22
  • 23. Singular solution does not exist. General solution is found out as usual. Type C: Equations of the form f ( x m z k p, y n z k q ) = 0 , where m, n ≠ 1; k ≠ −1 We make the transformations X = x 1− m , Y = y 1−n and Z = z k +1 ∂Z dZ ∂z dx Then P= = . . ∂X dz ∂x dX xm = ( k + 1) z p. k 1− m yn and Q = ( k + 1) z q. k 1− n Therefore the given equation reduces to  1 − m   1 − n   f   P,  Q  = 0  k + 1   k + 1   This is of type 1 equation. Problem:  p2 q2  z2 2 + 2 (1)Solve   =1  x y  Solution: 2 p q2  2 Given: z  2 + 2  = 1 x  y  It can be rewritten as (x −1 zp ) + ( y −1 zq ) = 1 2 2 ………..(1) which is of the form ( x m z k p ) + ( y n z k q ) = 1 2 2 we make the transformations X = x 1− m , Y = y 1−n and Z = z k +1 i.e. X = x 2 , Y = y 2 and Z = z 2 ∂z dz ∂Z dX 1 Then p= = . . = .P.2 x ∂x dZ ∂X dx 2 z ⇒ P = x −1 zp , 23
  • 24. Similarly, Q = y −1 zq , Using these in (1),it becomes P2 + Q2 = 1 …………(2) As (2) contains only P and Q explicitly, a solution of the equation will be of the form Z = aX + bY + c ………….(3) Therefore P = a and Q = b, obtained from (3) satisfy equation (2) a 2 + b 2 = 1, i.e. ⇒b = ± 1− a2 Therefore the complete solution of (2) is Z = aX ± 1 − a 2 Y + c Therefore the complete solution of (1) is z 2 = ax 2 ± 1 − a 2 y 2 + c Singular solution does not exist. General solution is found out as usual. Type D:  px qy  Equation of the form f  ,  = 0  z z  By putting X = log x, Y = log y and Z = log z the equation reduces to f ( P, Q ) = 0, ∂Z ∂Z where P = and Q = . ∂X ∂Y Problems: (1)Solve pqxy = z 2 . Solution: Given: pqxy = z 2 . …………….(1) Rewriting (1),  px  qy     = 1 . …………….(2)  z  z  24
  • 25.  px   qy  As (2) contains   and   , we make the substitutions  z   z  X = log x, Y = log y and Z = log z ∂z dz ∂Z dX 1 Then P= = . . = z.P. ∂x dZ ∂X dx x px i.e. =P z qx Similarly, =Q z Using these in (2), it becomes PQ = 1 …………..(3) which contains only P and Q explicitly. A solution of (3) is of the form Z = aX + bY + c …………(4) Therefore P = a and Q = b, obtained from (4) satisfy equation (3) 1 i.e. ab = 1 or b = a 1 Therefore the complete solution of (3) is Z = aX + Y + c a 1 Therefore the complete solution of (1) is log z = a log x + log y + c ……..(5) a General solution of (1) is obtained as usual. General solution of partial differential equations: Partial differential equations, for which the general solution can be obtained directly, can be divided in to the following three categories. (1) Equations that can be solved by direct (partial) integration. (2) Lagrange’s linear equation of the first order. (3) Linear partial differential equations of higher order with constant coefficients. Equations that can be solved by direct (partial) integration: Problems: 25
  • 26. ∂ 2u ∂u (1)Solve the equation = e −t cos x, if u = 0 when t = 0 and = 0 when x = 0. ∂x∂t ∂t Also show that u → sin x, when t → ∞ . Solution: ∂ 2u Given: = e −t cos x, …………….(1) ∂x∂t Integrating (1) partially with respect to x, ∂u = e −t sin x + f ( t ) …………….(2) ∂t ∂u When t = 0 and = 0 in (2), we get f ( t ) = 0 . ∂t ∂u Equation (2) becomes = e −t sin x …………….(3) ∂t Integrating (3) partially with respect to t, we get u = −e −t sin x + g ( x ) ……………(4) Using the given condition, namely u = 0 when t = 0 , we get 0 = − sin x + g ( x ) or g ( x ) = sin x Using this value in (4), the required particular solution of (1) is u = sin x (1 − e − t ) Now t →∞ { lim( u ) = sin x lim(1 − e −t ) t →∞ } = sin x i.e. when t → ∞, u → sin x . ∂z ∂z (2) Solve the equation = 3 x − y and = − x + cos y simultaneously. ∂x ∂y Solution: Given ∂z = 3x − y ................(1) ∂x ∂z = − x + cos y ..................(2) ∂y Integrating (1) partially with respect to x, 26
  • 27. 3x 2 z= − yx + f ( y ) …………..(3) 2 Differentiating (1) partially with respect to y, ∂z = − x + f ′( y ) …………...(4) ∂y Comparing (2) and (4), we get f ′( y ) = cos y f ( y ) = sin y + c ...................(5) Therefore the required solution is 3x 2 z= − yx + sin y + c , where c is an arbitrary constant. 2 Lagrange’s linear equation of the first order: A linear partial differential equation of the first order , which is of the form Pp + Qq = R where P, Q, R are functions of x, y , z is called Lagrange’s linear equation. working rule to solve Pp + Qq = R (1)To solve Pp + Qq = R , we form the corresponding subsidiary simultaneous equations dx dy dz = = . P Q R (2)Solving these equations, we get two independent solutions u = a and v = b . (3)Then the required general solution is f ( u , v ) = 0 or u = φ ( v ) or v = ψ ( u ) . dx dy dz Solution of the simultaneous equations = = . P Q R Methods of grouping: By grouping any two of three ratios, it may be possible to get an ordinary differential equation containing only two variables, eventhough P;Q;R are in general, functions of x,y,z. By solving this equation, we can get a solution of the simultaneous equations. By this method, we may be able to get two independent solutions, by using different groupings. 27
  • 28. Methods of multipliers: If we can find a set of three quantities l,m,n which may be constants or functions of the variables x,y,z, such that lP + mQ + nR = 0 , then the solution of the simultaneous equation is found out as follows. dx dy dz ldx + mdy + ndz = = = P Q R lP + mQ + nR Since lP + mQ + nR = 0, ldx + mdy + ndz = 0. If ldx + mdy + ndz = 0 is an exact differential of some function u ( x, y , z ) , then we get du = 0. Integrating this, we get dx dy dz u = a , which is a solution of = = . P Q R Similarly, if we can find another set of independent multipliers l ′, m′, n ′, we can get another independent solution v = b . Problems: (1)Solve xp + yq = x . Solution: Given: xp + yq = x . This is of Lagrange’s type of PDE where P = x, Q = y, R = x . dx dy dz The subsidiary equations are = = . x y x dx dy Taking first two members = x y Integrating we get log x = log y + log c1 x = c1 y i.e. ………….(1) x u= y dx dz Taking first and last members = . x x i.e. dx = dz . x − z = c2 Integrating we get ………......(2) v = x−z 28
  • 29. x  Therefore the solution of the given PDE is φ ( u , v ) = 0 i.e. φ  , x − z  = 0 . y    (2)Solve the equation ( x − 2 z ) p + ( 2 z − y ) q = y − x. Solution: Given: ( x − 2 z ) p + ( 2 z − y ) q = y − x. This is of Lagrange’s type of PDE where P = x − 2 z , Q = 2 z − y, R = y − x . dx dy dz The subsidiary equations are = = . ……….(1) x − 2z 2z − y y − x dx + dy + dz Using the multipliers 1,1,1, each ratio in (1)= . 0 dx + dy + dz = 0 . Integrating, we get x+ y+z =a ……………(2) ydx + xdy + 2zdz Using the multipliers y,x,2z, each ratio in (1)= . 0 d ( xy ) + 2 zdz = 0 . Integrating, we get xy + z 2 = b ……………(3) Therefore the general solution of the given equation is f ( x + y + z, xy + z 2 ) = 0 . (3)Show that the integral surface of the PDE x( y 2 + z ) p − y ( x 2 + z ) q = ( x 2 − y 2 ) z . Which contains the straight line x + y = 0, z = 1 is x 2 + y 2 + 2 xyz − 2 z + 2 = 0 . Solution: The subsidiary equations of the given Lagrange ‘s equation are dx dy dz = = 2 . x( y + z ) − y ( x + z ) ( x − y 2 ) z 2 2 ……………(1) 1 1 1 1 1 1 dx + dy + dz Using the multipliers , , , . each ratio in (1)= x y z . x y z 0 1 1 1 dx + dy + dz = 0 . x y z Integrating, we get xyz = a ……………(2) 29
  • 30. xdx + ydy − dz Using the multipliers y,x,-1, each ratio in (1)= . 0 xdx + ydy − dz = 0 . Integrating, we get x 2 + y 2 − 2 z = b ……………(3) The required surface has to pass through x+ y =0 ……………(4) z =1 Using (4) in (2) and (3), we have − x2 = a ……………(5) 2x 2 − 2 = b Eliminating x in (5) we get, 2a + b + 2 = 0 …………….(6) Substituting for a and b from (2) and (3) in (6), we get x 2 + y 2 + 2 xyz − 2 z + 2 = 0 , which is the equation of the required surface. Linear P.D.E.S of higher order with constant coefficients: The standard form of a homogeneous linear partial differential equation of the n th order with constant coefficients is ∂nz ∂nz ∂nz ∂n z a 0 n + a1 n −1 + a 2 n −2 2 + ... + a n n = R ( x, y ) …………….(1) ∂x ∂x ∂y ∂x ∂y ∂y where a’s are constants. ∂ ∂ If we use the operators D ≡ and D′ ≡ , we can symbolically write equation (1) ∂x ∂y as (a D 0 n + a1 D n −1 D ′ + ... + a n D ′ n ) z = R( x, y ) …………….(2) f ( D, D ′) z = R ( x, y ) …………….(3) where f ( D, D ′) is a homogeneous polynomial of the n th degree in D and D ′ . The method of solving (3) is similar to that of solving ordinary linear differential equations with constant coefficients. 30
  • 31. The general solution of (3) is of the form z = (complementary function)+(particular integral),where the complementary function is the R.H.S of the general solution of 1 f ( D, D ′) z = 0 and the particular integral is given symbolically by R ( x, y ) . f ( D, D ′) Complementary function of f ( D, D ′) z = R( x, y ) : C.F of the solution of f ( D, D ′) z = R( x, y ) is the R.H.S of the solution of f ( D, D ′) z = 0 . …………(1) In this equation, we put D = m, D ′ = 1 ,then we get an equation which is called the auxiliary equation. Hence the auxiliary equation of (1) is a 0 m n + a1 m n −1 + ... + a n = 0 ………….(2) Let the roots of this equation be m1 , m2 ,...mn . Case 1: The roots of (2) are real and distinct. The general solution is given by z = φ1 ( y + m1 x ) + φ 2 ( y + m2 x ) + ... + φ n ( y + mn x ) Case 2: Two of the roots of (2) are equal and others are distinct. The general solution is given by z = φ1 ( y + m1 x ) + φ 2 ( y + m1 x ) + ... + φ n ( y + mn x ) Case 3: ‘r’ of the roots of (2) are equal and others distinct. The general solution is given by z = φ1 ( y + m1 x ) + xφ 2 ( y + m1 x ) + ... + x r −1φ r ( y + mr x ) To find particular integral: Rule (1): If the R.H.S of a given PDE is f ( x, y ) = e ax +by , then 1 P.I = e ax +by f ( D, D ′ ) Put D = a, D ′ = b 31
  • 32. 1 P.I = e ax +by if f ( a, b ) ≠ 0 f ( a, b ) If f ( a, b ) = 0, refer to Rule (4). Rule (2): If the R.H.S of a given PDE is f ( x, y ) = sin ( ax + by ) or cos( ax + by ) , then 1 P.I = sin ( ax + by ) or cos( ax + by ) f ( D, D ′ ) Replace D 2 = −a 2 , D ′ 2 = −b 2 and DD ′ = −ab in f ( D, D ′) provided the denominator is not equal to zero. If the denominator is zero, refer to Rule (4). Rule (3): If the R.H.S of a given PDE is f ( x, y ) = x m y n , then 1 P.I = xm yn f ( D, D ′ ) = { f ( D, D ′ ) } −1 (x m , yn ) Expand { f ( D, D ′)} −1 by using Binomial Theorem and then operate on x m y n . Rule (4): If the R.H.S of a given PDE f ( x, y ) is any other function [other than Rule(1), (2) and(3)] resolve f ( D, D ′) into linear factors say ( D − m1 D ′)( D − m2 D ′) etc. then the 1 P.I = f ( x, y ) ( D − m1 D ′)( D − m2 D ′) Note: If the denominator is zero in Rule (1) and (2) then apply Rule (4). Working rule to find P.I when denominator is zero in Rule (1) and Rule (2). If the R.H.S of a given PDE is of the form sin ( ax + by ) or cos( ax + by ) or e ax +by 1 xn Then P.I = . f ( ax + by ) = n f ( ax + by ) ( bD − aD ′) b n! This rule can be applied only for equal roots. Problems: (1) Solve ( D 3 − 3DD ′ 2 + 2 D ′ 3 ) z = e 2 x − y + e x + y Solution: Given: ( D 3 − 3DD ′ 2 + 2 D ′ 3 ) z = e 2 x − y + e x + y 32
  • 33. The auxiliary equation is m 3 − 3m + 2 = 0 ⇒ m = 1,1,−2 C.F = xf 1 ( y + x ) + f 2 ( y + x ) + f 3 ( y − 2 x ) P.I = 1 (e 2 x− y + e x+ y ) D − 3D D ′ + 2 D ′ 3 22 3 1 1 = e 2 x− y + e x+ y ( D + 2 D ′)( D − D ′) 2 ( D − D ′) ( D + 2 D ′ ) 2 1 1 1 1 = . e 2 x− y + . e x+ y 9 D + 2D ′ 9 ( D − D ′) 2 1  2 x− y x 2 x+ y  =  xe + e  9 2  The general solution of the given equation is x 2 x− y x 2 x+ y z = xf1 ( y + x ) + f 2 ( y + x ) + f 3 ( y − 2 x ) + e + e 9 18 (2)Solve ( D 2 + 4 DD ′ − 5 D ′ 2 ) z = xy + sin ( 2 x + 3 y ) Solution: Given: ( D 2 + 4 DD ′ − 5 D ′ 2 ) z = xy + sin ( 2 x + 3 y ) The auxiliary equation is m 2 + 4m − 5 = 0 ⇒ m = 1,−5 C.F = φ1 ( y − 5 x ) + φ 2 ( y + x ) 1 ( P.I ) 1 = ( xy ) D + 4 DD ′ − 5 D ′ 2 2 1 = ( xy ) 2 D′  D 1 + 2 ( 4 D − 5 D ′)   D  −1 1  D′  = 2 1 + 2 ( 4 D − 5 D ′)  ( xy ) D  D  1  D′  = 2 1 − 2 ( 4 D − 5D ′) + ...( xy ) D  D  33
  • 34. 1 = 2 ( xy ) − 14 .4 D ′( xy ) D D 3 x y 1 = − 4 ( 4x) 6 D 1 3 1 5 = x y− x 6 30 1 ( P.I ) 2 = sin ( 2 x + 3 y ) D + 4 DD ′ − 5 D ′ 2 2 1 = sin ( 2 x + 3 y ) ( − 2) + 4.( − 2.3) − 5( − 3) 2 2 1 = sin ( 2 x + 3 y ) 17 Therefore the general solution is z = φ1 ( y − 5 x ) + φ 2 ( y + x ) + 6 ( x y ) − 30 x 5 + 17 sin ( 2 x + 3 y ) 1 3 1 1 (3)Solve ( D 2 − 2 DD ′ + D ′ 2 ) z = x 2 y 2 e x + y Solution: Given: ( D 2 − 2 DD ′ + D ′ 2 ) z = x 2 y 2 e x + y The auxiliary equation is m 2 − 2m + 1 = 0 ⇒ m = 1,1 C.F = xf 1 ( y + x ) + f 2 ( y + x ) e x+ y ( x 2 y 2 ) 1 P.I = ( D − D ′) 2 1 = e x+ y x2 y2 { ( D + 1) − ( D ′ + 1)} 2 1 = e x+ y x2 y2 ( D − D ′) 2 −2 1  D′  1 −  ( x y ) x+ y =e 2 2 D2  D 1  2 D′ D′ 2  = e x + y 2 1 + + 3 2 ( x 2 y 2 ) D  D D   34
  • 35. x y + ( 2 x y ) + 2 ( 2 x ) 1  2 2 2 3 2  = e x+ y 2 D2  D D  = e x + y  y 2 2 ( x 2 ) + 4 y. 3 ( x 2 ) + 6. 4 ( x 2 )   1 1 1   D D D  1 1 1 6  x+ y =  x4 y 2 + x5 y + x e  12 15 60  Therefore the general solution is 1 1 1 2  4 x+ y z = xf1 ( y + x ) + f 2 ( y + x ) +  y 2 + xy + x x e  12 15 60  35
  • 36. UNIT 1 Part A (1)Form a partial differential equation by eliminating arbitrary constants a and b from z = ( x + a ) + ( y + b) 2 2 Ans: Given z = ( x + a ) 2 + ( y + b ) 2 ……. (1) ∂z p= = 2( x + a )  ( 2) ∂x ∂z q= = 2( y + b )  ( 3) ∂y p2 q2 Substituting (2) & (3) in (1), we get z = + 4 4 (2) Solve: ( D 2 − 2 DD ′ + D 2 ) z = 0 Ans: Auxiliary equation m 2 − 2m + 1 = 0 ( m − 1) 2 = 0 m = 1,1 z = f 1 ( y + x ) + xf 2 ( y + x ) (3)Form a partial differential equation by eliminating the arbitrary constants a and b from the equation ( x − a ) 2 + ( y − b ) 2 = z 2 cot 2 α . Ans: Given: ( x − a ) 2 + ( y − b ) 2 = z 2 cot 2 α ….. (1) Partially differentiating with respect to ‘x’ and ‘y’ we get 2( x − a ) = 2 zp cot 2 α ….. (2) 2( y − b ) = 2 zq cot α 2 …... (3) (2) ⇒ x − a = zp cot 2 α …… (4) (3) ⇒ y − b = zq cot 2 α ….. (5) Substituting (4) and (5) in (1) we get z 2 p 2 cot 4 α + z 2 q 2 cot 4 α = z 2 cot 2 α . ⇒ p 2 + q 2 = tan 2 α . (4)Find the complete solution of the partial differential equation p 2 + q 2 − 4 pq = 0. Ans: Given: p 2 + q 2 − 4 pq = 0. …….. (1) Let us assume that z = ax + by + c ……… (2) 36
  • 37. be the solution of (1) Partially differentiating with respect to ‘x’ and ‘y’ we get ∂z p= =a ∂x …….. (3) ∂z q= =b ∂y Substituting (3) in (1) we get a 2 + b 2 − 4ab = 0 From the above equation we get, 4b ± 16b 2 − 4a 2 b 2 a= 2 a = b ± b 4 − a2 ………. (4) Substituting (5) in (2) we get ( ) z = b 1 ± 4 − a 2 x + by + c (5)Find the PDE of all planes having equal intercepts on the x and y axis. Ans: The equation of such plane is x y z + + =1 ………. (1) a a b Partially differentiating (1) with respect to ‘x’ and ‘y’ we get 1 p b + =0⇒ p =− ……….. (2) a b a 1 q b + =0⇒q =− ……….. (3) a b a From (2) and (3), we get p=q (6)Find the solution of px 2 + qy 2 = z 2 . Ans: The S.E is dx dy dz = = x2 y2 z 2 Taking first two members, we get dx dy = x2 y2 Integrating we get −1 −1 = + c1 x y 1 1 i.e u  −  = c1  y x   Taking last two members, we get 37
  • 38. dy dz = y2 z2 Integrating we get −1 −1 = + c2 y z 1 1 i.e v −  = c 2 z y   The complete solution is 1 1 1 1 φ − , −  = 0  y x z y   (7)Find the singular integral of the partial differential equation z = px + qy + p 2 − q 2 . Ans: The complete integral is z = ax + by + a 2 − b 2 . ∂z x = x + 2a = 0 ⇒ a = − ∂a 2 ∂z y = y − 2b = 0 ⇒ b = ∂b 2 Therefore x2 y2 x2 y2 x2 y2 z=− + + − =− + 2 2 4 4 4 4 ⇒ y − x = 4z 2 2 (8)Solve: 2 p + q2 = m2. Ans: Given p2 + q2 = m2 …….. (1) Let us assume that z = ax + by + c …… (2) be the solution of (1) Partially differentiating with respect to ‘x’ and ‘y’ we get ∂z p= =a ∂x …….. (3) ∂z q= =b ∂y Substituting (3) in (1) we get a2 + b2 = m2 This is the required solution. 38
  • 39. (9)Form a partial differential equation by eliminating the arbitrary constants a and b from z = ax n + by n . Ans: Given z = ax n + by n . ……… (1) Partially differentiating with respect to ‘x’ and ‘y’ we get ∂z p p= = a.nx n −1 ⇒ a = n −1 ∂x nx …….. (2) ∂z n −1 q q= = b.ny ⇒ b = n −1 ∂y ny Substituting (2) in (1) we get p q z = n −1 x n + n −1 y n nx ny 1 z = ( px + qy ) n This is the required PDE. (10)Solve: (D 3 + D 2 D ′ + DD ′ 2 + D ′ 3 ) z = 0. Ans: Auxiliary equation m3 + m2 + m + 1 = 0 ( m + 1) 3 = 0 m = −1,−1,−1 z = f 1 ( y − x ) + xf 2 ( y − x ) + x 2 f 3 ( y − x ) (11)Form a partial differential equation by eliminating the arbitrary constants a and b from z = ( x 2 + a 2 )( y 2 + b 2 ). Ans: Given z = ( x 2 + a 2 )( y 2 + b 2 ) ……. (1) ∂z = 2 x( y 2 + b 2 ) ⇒ y 2 + b 2 = p p=  ( 2 ) ∂x 2x ∂z = 2 y( x 2 + a 2 ) ⇒ x 2 + a 2 = q q=  ( 3) ∂y 2y q p Substituting (2) & (3) in (1), we get z = . 2 y 2x pq = 4 xyz (12)Solve: ( D 2 − DD′ + D′ − 1) z = 0 Ans: The given equation can be written as 39
  • 40. ( D − 1)( D − D ′ + 1) z = 0 We know that the C.F corresponding to the factors ( D − m1 D ′ − α 1 )( D − m2 D′ − α 2 ) z = 0 is z = e α1x f 1 ( y + m1 x ) + e α 2 x f 2 ( y + m2 x ) In our problem α 1 = 1, α 2 = −1, m1 = 0, m2 = 1 C.F = e x f 1 ( y + x ) + e − x f 2 ( y + x ) z = e x f1 ( y + x ) + e − x f 2 ( y + x ) (13)Form a partial differential equation by eliminate the arbitrary function f from  xy  z = f  .  z  Ans:  xy  Given : z = f  .  z   xy  zy − xy. p p = f ′ .. ............(1)  z  z2  xy  zx − xy.q q = f ′ .. ............(2)  z  z2 From (1), we get  xy  pz 2 f ′  = .............(3)  z  zy − xyp Substituting (3) in(2), we get pz 2 zy − xy. p p= . zy − xyp z2 (14)Solve: (D 3 + 2 D 2 D ′ − DD ′ 2 − 2 D ′ 3 ) z = 0. Ans: Auxiliary equation m 3 + 2m 2 − m − 2 = 0 m = 1,−1,−2 Solution is z = f 1 ( y + x ) + f 2 ( y − x ) + f 3 ( y − 2 x ) (15)Obtain partial differential equation by eliminating arbitrary constants a and b from ( x + a ) 2 + ( y + b) 2 + z 2 = 1. Ans: Given ( x − a ) 2 + ( y − b ) 2 + z 2 = 1 ……. (1) 40
  • 41. 2( x − a ) + 2 zp = 0 ⇒ ( x − a ) = − zp .........(2) 2( y − b ) + 2 zq = 0 ⇒ ( y − b ) = − zq  ( 3) Substituting (2) & (3) in (1), we get z 2 p2 + z 2q2 + z 2 = 1 1 p2 + q2 +1 = z2 (16)Find the general solution of ∂2 z ∂2 z ∂2 z 4 2 − 12 + 9 2 = 0. ∂y ∂x∂y ∂ y Ans: Auxiliary equation is 4m 2 − 12m + 9 = 0 12 ± 144 − 144 m= 8 3 3 m= , 2 2 General solution is  3   3  z = f 1  y + x  + xf 2  y + x   2   2  (17)Find the complete integral of ∂z ∂z p + q = pq, where p= ,q = . ∂x ∂y Ans: Let us assume that z = ax + by + c ……… (1) be the solution of the given equation. Partially differentiating with respect to ‘x’ and ‘y’ we get ∂z p= =a ∂x …….. (2) ∂z q= =b ∂y Substituting (2) in (1) we get a a + b = ab ⇒ b = a −1 Substituting the above in (1) we get  a  z = ax +  y + c  a −1 This gives the complete integral. (18)Solve: 41
  • 42. (D 3 − 3 DD ′ 2 + 2 D ′ 3 ) z = 0 Ans: Auxiliary equation m 3 − 3m + 2 = 0 m = 1,1,−2 Solution is z = f 1 ( y + x ) + xf 2 ( y + x ) + f 3 ( y − 2 x ) (19)Find the PDE of the family of spheres having their centers on the line x=y=z. Ans: The equation of such sphere is ( x − a) 2 + ( y − a) 2 + ( z − a) 2 = r 2 Partially differentiating with respect to ‘x’ and ‘y’ we get 2( x − a ) + 2( z − a ) p = 0 ...........(1) 2( y − a ) + 2( z − a ) q = 0 ...........(2) From (1), x + zp a= ...........(3) 1+ p From (2), y + zq a= ...........(4) 1+ q From (3) and (4), we get x + zp y + zq = 1+ p 1+ q This is the required PDE. (20)Solve: ∂3 z ∂2z ∂3z ∂3 z −2 2 −4 + 8 3 = 0. ∂x 3 ∂x ∂y ∂x∂y 2 ∂y Ans: Auxiliary equation m 3 − 2m 2 − 4 m + 8 = 0 m = 2,2,−2 Solution is z = f 1 ( y + 2 x ) + xf 2 ( y + 2 x ) + f 3 ( y − 2 x ) Part B (1)(i) Form a partial differential equation by eliminating arbitrary functions from z = xf ( 2 x + y ) + g ( 2 x + y ) (ii) Solve: p 2 y (1 + x 2 ) = qx 2 (2)(i) Solve: x( z 2 − y 2 ) p + y ( x 2 − z 2 ) q = z ( y 2 − x 2 ) 42
  • 43. ∂3z ∂3 z (ii) Solve: 3 − 2 2 = e x + 2 y + 4 sin ( x + y ) ∂x ∂x ∂y (3)(i) Solve: ( x 2 − y 2 − z 2 ) p + 2 xyq − 2 xz = 0 (ii) Solve: ( D 2 − DD ′ − 2 D ′ 2 ) z = 2 x + 3 y + e 3 x + 4 y (4)(i) Solve: z 2 ( p 2 + q 2 ) = x 2 + y 2 (ii) Solve: ( D 2 + 3DD ′ − 4 D ′ 2 ) z = sin y (5)(i) Solve: ( x 2 + y 2 + yz ) p + ( x 2 + y 2 − xz ) q = z ( x + y ) (ii) Solve: ( D 2 − DD ′ − 20 D ′ 2 ) z = e 5 x + y + sin ( 4 x − y ) (6)(i) Solve: z = p 2 + q 2 (ii) Solve: ( D 2 + DD ′ − 6 D ′ 2 ) z = x 2 y + e 3 x + y (7)(i) Solve: ( y 2 + z 2 ) p − xyq + xz = 0 (ii) Solve: ( D 2 − 6 DD ′ + 5D ′ 2 ) z = xy + e x sinh y (8)(i) Solve: p (1 − q 2 ) = q (1 − z ) (ii) Solve: ( D 2 − 4 DD ′ + 4 D ′ 2 ) z = xy + e 2 x + y (9)(i) Solve: ( x 2 − yz ) p + ( y 2 − zx ) q = z 2 − xy (ii) Solve: x( y − z ) p + y ( z − x ) q = z ( x − y ) (10)(i) Solve: p 2 + x 2 y 2 q 2 = x 2 z 2 (ii)Solve: ( D 2 − D ′ 2 − 3D + 3D ′) z = xy (11)(i) Form the partial differential equation by eliminating f and φ from z = f ( y) + φ( x + y + z) (ii) Solve: ( D ′ 2 − 2 DD ′) z = x 3 y + e 2 x (12)(i) Find the complete integral of p + q = x + y (ii) Solve: y 2 p − xyq = x( z − 2 y ) (13)(i) Solve: ( 3 z − 4 y ) p + ( 4 x − 2 z ) q = 2 y − 3 x (ii) Solve: ( D 2 − 2 DD ′ − D ′ 2 + 3D + 3D ′ + 2 ) z = ( e 3 x + 2e −2 y ) 2 (iii) Solve: ( D 2 + 4 DD ′ − 5 D ′ 2 ) z = sin ( x − 2 y ) + e 2 x − y (14)(i) Solve z 2 = 1 + p 2 + q 2 (ii) Solve: ( y − z ) p − ( 2 x + y ) q = 2 x + z 43
  • 44. (15)(i) Form a partial differential equation by eliminating arbitrary functions f and g in z = x 2 f ( y ) + y 2 g ( x) (ii) Solve: ( D 2 − DD ′ − 20 D ′ 2 ) z = sin ( 4 x − y ) + e 5 x + y (16)(i) Form a partial differential equation by eliminating arbitrary functions f and g in z = f ( x3 + 2 y) + g( x3 − 2 y) (ii) Solve: ( y − xy ) p + ( yz − x ) q = ( x + y )( x − y ) (17)(i) Find the singular solution of z = px + qy + p2 + q2 +1 (ii) Solve: ( D 2 − DD ′ − 30 D ′ 2 ) z = xy + e 6 x + y (18) (i) Solve: ( D 3 − 7 DD ′ 2 − 6 D ′ 3 ) z = sin ( x + 2 y ) + e 2 x + y (ii) Find the singular integral of a partial differential equation z = px + qy + p 2 − q 2 (19)(i) Solve: ( 4 D 2 − 4 DD ′ + D ′ 2 ) z = 16 log( x + 2 y ) (ii) Form a partial differential equation by eliminating arbitrary functions ‘f’ from f ( z − xy, x 2 + y 2 ) = 0 (20)(i) Solve: ( D 2 + D ′ 2 ) z = sin 2 x sin 3 y + 2 sin 2 ( x + y ) (ii) Solve: p 2 + x 2 y 2 q 2 = x 2 z 2 44